AMA | July 2022

Welcome to the July 2022 Ask Me Anything episode of Mindscape! These monthly excursions are funded by Patreon supporters (who are also the ones asking the questions). We take questions asked by Patreons, whittle them down to a more manageable number -- based primarily on whether I have anything interesting to say about them, not whether the questions themselves are good -- and sometimes group them together if they are about a similar topic.

Big news this week! Mindscape is working with Bold.org to sponsor a college scholarship for students interested in studying the fundamental nature of reality. Listeners can find more details and donate here. Our immediate goal is to raise $10,000, and I will match the first $5,000, so this shouldn't be too hard for us here. Hopefully we can raise much more! And hopefully this will help encourage someone who might not otherwise have been able to study this kind of topic.

Support Mindscape on Patreon.

July AMA questions

https://bold.org/funds/mindscape/

Edward A. Morris
In your discussion with Ed Yong about animal sensation, I noticed you never used the word qualia. Is this because you consider the whole concept of qualia, including questions like whether one organism's color spectrum might be inverted relative to another's, inherently nonsensical in a poetic-naturalist understanding of consciousness? Or would the language of qualia be something like the language of free will, a legitimate way of referring to a concept that emerges in higher-level descriptions of consciousness?

Rob Patro
You've previously done several episodes on representative government, democracy, and how these come to an end. Given the recent devastating Supreme Court rulings overturning on Roe v. Wade, stripping states of the rights to impose reasonable gun laws, preventing judicial review of partisan gerrymandering, etc., what do you see as the likelihood that we are already in a failed democracy and have just not fully realized it yet? If not, what actions can we really take to address this decline apart from voting; as the persistent attacks on voting rights and fair local election rules seem targeted to minimize the efficacy of this most direct and fundamental response?

Peter Blankenheim
The BaBar group using the Stanford Linear Acccelerator recently observed that the B meson's oscillation to its anti particle takes less time than the reverse oscillation, violating T symmetry. When that got called an "Arrow of Time" proof, you objected, saying that the Arrow of Time comes from the low entropy of the Big Bang, and said "indeed, the entire phenomenon of T violation — has absolutely nothing to do with that arrow of time."
But wait, a reaction that is faster than its reverse IS pretty special. Why doesn't that qualify as an Arrow of Time?

***********************************
lēma
I've been wanting to ask for a while what your actual take on Stephen Wolfram's hypergraph project is.

Brandon Lewis
I see a lot of overlap between how you describe quantum gravity in your early episode on the topic and Stephen Wolfram's graph-theoretic approach to fundamental physics. Yet I was left with the impression that you don't find Wolfram's approach all that compelling. Could you expand a bit more on this topic? Do you feel it's too "computer sciency" or are you skeptical of his claims to have recovered QM from his new approach? Or do you just not feel that it will lead to any new insight? Or do I just have the wrong impression?
*************************************

Paul Hess
Thank you for Solo episode #200. In that episode you mentioned that if the cosmological constant stays constant, the universe would expand into something "approaching DeSitter space". You then described this as a near perfect black body, with a defined temperature, that lasts for an infinite time.
Why should the universe ever stop expanding or cooling? Why would it remain in any stable state with any defined temperature instead of continually expanding and cooling?

Tom Hawkins
Black holes have been proposed to be the source of some (if not all) of the dark matter. Your comment?

Andrew Goldstein
Respect for data generated by and conclusions drawn from scientifically sound research appears to be in decline, especially when it contradicts held beliefs. How do you think the rest of us "science believers" can help facilitate more people to listen to what science has to say about everything from vaccines to climate change to emergence?

Anonymous
I saw a PBS Spacetime video where Matt O'Dowd explores whether it's possible that the Big Bang is a time-reversed black hole (a white hole). My question is, supposing that it's possible for a white hole to have black holes inside of its event horizon, would it also be possible for a black hole to have white holes inside of its event horizon?

Louis B.
I am curious about the past hypothesis' implications on our ability to rigorously study the past hypothesis. You seemed to indicate at times that even when trying to scientifically study the entropy of the early universe the mere process of making observations and drawing inferences itself must already presume the past hypothesis. It would seem we are forced into at least some form of circular reasoning while trying to conduct science on the question of the entropy of the early universe?

Sid Huff
In listening to your Mindscape interviews, I’ve noticed that many of your guests have recently published a book. Furthermore, it seems clear from the interviews that you have almost always READ and absorbed their book prior to the interview. Is reading these books included in the one-day-per-week you have mentioned that you spend on the podcast?

f_h
Just to make things clear in my head…: when you say that a universe can fluctuate into existence (I think you said that…), does this really mean that fluctuations in De Sitter space can cause elementary particles to randomly come together in a configuration that creates an entire universe, just like ours?
If so, isn’t this then the answer to the question about the origin of our universe? How can we disprove that it wasn’t created this way?

Trevor Villwock
PRIORITY QUESTION: Shouldn't we consider the default, "rational" view of an afterlife to be that life repeats eternally rather than simply "ending" at your death?
[More explanation]

Carlos Nunez
Between the view of Francis Fukuyama that the world will progress towards a global capitalist democratic order and Samuel Huntington's view of warring civilizations in perpetual conflict due to profound cultural differences, which position seems more reasonable to you? Can non-Western civilizations, such as China, ever embrace Western democratic values?

***********************************
Justin Wolcott
The fine tuning argument says that the odds that the conditions of a universe that supports life randomly occurring is 10^-223; but the odds that god would create a universe that supports life is 100%. Therefore god is exponentially more probable than no god. The fine tuning argument itself feels flawed, by picking an arbitrary phenomenon (life) and applying a success criterion for a pet theory after the fact.
I can't put my finger on it but this feels like a dishonest research technique like p-hacking. Is it?

Mark Scheuern
I’m currently reading David J. Chalmer’s Reality + and enjoying it. I came across a statement, though, that strikes me as incorrect and I’m curious about your thoughts on it. He says that a weakness to the multiverse as a solution to the fine-tuning problem is that it only bumps the problem up a level because the multiverse would have to be fine-tuned to allow for universes suitable for life to come into being. I am under the impression that all we need are string theory and inflation and we end up with universes with a variety of physical laws and we don’t need some set of laws for the multiverse that determine how many and what sorts of universes we get. Am I thinking about this incorrectly?
************************************

Bruno Teixeira
There have been some news about a "new particle discovered", the "Axial Higgs Boson" or the "Magnetic Higgs". I read the abstract of the article but it was out of my league. Can we get your take on it?

Nicholas Chapman
Boltzmann's formula for entropy gives entropy as proportional to the log of the number of microstates for a given macrostate. Using this definition, the second law of thermodynamics then just becomes almost a tautology - that more likely states are (overwhelmingly) more likely to occur in the future. How then is the second law useful in any way when defined like this?

richard keenan
"PRIORITY QUESTION"
Do you think it is acceptable to come to conclusions about other political and sociological situations, like those in China for example, without conducting the proper research and having evidence to back up most peoples claim, that a liberal democracy is the only civilised way to govern a population?
It seems to me that almost everyone gets their information on places like China from mainstream media, who are known to lie, and to be biased in the extreme when it comes to China in particular. It is also know that the United States government has a budget of millions to fund anti Chinese propaganda, when surely that money could be better spent on US citizens welfare.

Yigal Kamel
Is it possible/reasonable to formulate a version of "many-worlds" QM without actually believing in the many worlds? In other words, using entanglement/decoherence to explain measurement/collapse and the associated probabilities as many-worlds does, but being agnostic about whether the other branches "exist".

Joye Colbeck
Does the CMB have a spectral signature?

Dom
Do you think physics can ever be finished?

Seumas MacLennan
Would love to hear any thoughts you might have on Karl Popper's tolerance paradox. Personally, I'm constantly frustrated by friends who are on the Liberal left, that constantly advocate for free speech, acknowledge that there is a line somewhere, but refuse to define where that line is!

Noble Gas
What are some ways to wrap our minds around the infinite regress problem as it pertains to the origin of the universe? It seems like an intractable problem because it's always possible to ask "what came before that?" If the current answer is that we just don't know, is it possible to say what you think an answer might look like?

krathorlucca
Can you explain what the main disagreements are that cause some people to believe quantum entanglement is local vs non-local? And is it correct that within many worlds QM is entirely local?

alexandra bates
Do you think the US constitution was a good basis for a democracy at the time of its writing? Do you think it remains a good basis for today’s industrial, technologically advanced US?

Vladimir Belykh
You, and hopefully most of us, listeners, are strongly in favor of democracy and want it to be preserved.
However, as a thought experiment, could you come up with some genuinely reasonable arguments *AGAINST* democracy? Arguments that you *personally* would find reasonable.

Anita Tomasik
How much credence do you give to the concept of Boltzmannian Immorality? Particularly, do those human brain-like fluctuations called Boltzmann’s brains, last long enough to give rise to any form of self-consciousness at all?

FlyingWaffle
I don't have an actual question today.
I asked GPT-3 to "Write a haiku in the style of Sean Carroll", here are some of its best answers:

He's a theoretical physicist
And he writes about science and math
He's also pretty funny

Chris
What would you say are the starting points for your worldview/philosophy/science that are themselves not based on something else? And how do you view/justify these? As the most self-consistent set of hypotheses or something else?

joey
I have been thinking at our society is really bad for people with serious mental illnesses like schizophrenia. As in we believe people are rational and give them a degree of self agency. But for people who behave irrationally many of our social safety nets breaks down. Other than a benevolent overload what do you think would make the world better.

Rob Butler
If macroscopic objects behave ‘classically’, where does the quantum world end and the macroscopic world begin?! If these 2 descriptions (small quantum particles and large macroscopic objects) act so differently in our reality, why do we need a unifying theory?

Brent Meeker
In Mindscape 63 | Solo: Finding Gravity Within Quantum Mechanics you speculated that there are only finitely many DoF in a given volume. Wouldn't this imply that there are also only finitely many possible states and so there would be a smallest non-zero probability of any possible event. Our present form of QM would be an effective approximation to this, but in the branching picture of Everettian QM, branches would be terminated when falling to a probability lower than this smallest non-zero probability. Due to the bazillion branches per second in unobserved "measurements", all but contrived isolated events, as in laboratories would evolve quasi-classically.

Greg
An omniscient Superbeing gives you one and only one question about the true physical nature of the universe which they will answer correctly, but it has to be a yes or no question. You know that the answer will be correct and absolutely true. What do you ask?

Paul Torek
In your talk "The Arrow of Time in Causal Networks" you say "If you want to draw causal network arrows that only go one way in time, you only get right answers drawing them from past to future." That's a pretty big IF. Is there more to say - what if someone doesn't want to do that?
(I simplified, hopefully not oversimplified, the quote to save time.)

Patricia Paulson
I watched your series of lectures "Dark Matter, Dark Energy: The Dark Side of the Universe" on The Great Courses/ Wondrium. They were recorded in 2007, I believe, before the LHC was operational and the Higgs was discovered; you mentioned the anticipation. I believe you said something like how you hoped in 5 years or so there would be some big discoveries regarding dark energy and dark matter. 15 years later, would you have lots more discoveries to expound upon in a new series of lectures or would it be pretty much the same mysteries as 2007?

phil
Quantum mechanics claims that one cannot predict with certainty when a radioactive atom will decay, even given a complete understanding of the laws of physics as well as complete knowledge of the state of the Universe. How can quantum mechanics make this claim given that we don’t yet have a complete understanding of the laws of physics nor complete knowledge of the state of the Universe?

Joel Curtis
Where do you stand on the merits of the rubber sheet analogy to illustrate spacetime curvature? I see lots of people denigrate the analogy, but I think it does a fair job of striking a balance between making the idea accessible to those without technical background and not sending people away with entirely the wrong idea.

Walter white
PRIORITY QUESTION : Does the block universe imply eternal life/immortality in "any" sense? If yes , does it mean our life repeat itself over and over again? Since the world lines are of finite length I couldn't imagine any other way in which we live forever OR am I misunderstanding the concept ?

Eric Dovigi
Possibly a dumb question, but I feel like this is a safe zone for that. Q: Why do objects bend spacetime? I get that they do, but I mean... *whyyyyy*?

Gary Upshaw
Throughout human history, humans have accessed new forms of energy sources: animal muscle, wood, wind, coal, oil, nuclear fission, solar. Nuclear fusion is on the horizon. With each phase, human population has increased, and Earth's resources have been accessed with greater efficiency, greater speed, and are either depleted or on the path to depletion: megafauna, soils, water, trees, fisheries, minerals, oil, and so on. Pollution and environmental degradation have ended past civilizations, and is threatening our planet today.The promise of near unlimited and cheap fusion energy will enable more and reliable sources of food, and enable the stripping of Earth resources at an increased intensity. Do you think development of cheap and abundant fusion energy will lead to these scenarios, and could such technological advancements be the cause of ending sentient life on extraterrestrial planets and explain why we have not yet detected alien intelligent life?

Richard Graff
I’m 62 and aware of the valuable perspective that comes with those and later years. However, I’ve come to believe our nation would benefit from an upper limit, somewhere before 60, on the age a person can be elected a U.S. president, senator, representative or continue to serve on the Supreme Court. What are your thoughts on this idea?

Mr. Foo
PRIORITY QUESTION: With relation to MWI, do the other worlds have a 'XYZ' coordinate within space? Or do they exist in another location that cannot be definied by distance from Earth?

Graham Clark
In his book The Revolt Of The Public, Martin Gurri talks a bit about science, and using climate science as an example, portrays the institution as acting like a cartel, defending its own researchers and findings against any opposing views, however valid, and of a peer-review process hopelessly inadequate in a time of massive data sets and esoteric research areas. Does any of this ring true to you, in your research areas or elsewhere?

Late Night Linux
The scientific method was devised by white men. Same with math. Some people say that makes them inherently biased and possibly even racist. Is this something that you've thought about?

Joe Moskowitz
I was watching your 'Something Deeply Hidden" talk at Google and something jiggled loose in my brain and Many Worlds suddenly seemed more reasonable. The slide with the expression for the dead and alive Schrodingers cat multiplied by the environment and the observer because of the universal wave function seems to have been the exact tipping point even though I've seen that before. I'm highly suspicious of 50 years of my opinions shifting in a few seconds. Any advice?

Simon Reynolds
Have you ever tried to enter a friendly wager with another scientist, and would you consider that as a good way to help people consider their positions more carefully?

P Walder
In your 2020 Royal Institution on the multiverse you explained how linear tracks that appear in cloud chambers only ‘seem’ to infer a particle-like cause rather than a wave-like cause. The talk was excellent and helped clarify what can and can’t be known about the multiverse but I’m still confused about why there are linear cloud chamber tracks. Could you have another go at explaining this phenomenon?

Daniel Fox
I recently had a minor stroke while running. No apparent lasting damage. However, no one can say exactly what caused the occlusion - it seems just random thing without any of the usual health or behavioral indicators. I've been an ultra distance
runner for years, and it's been a big part of my identity. The question is: how would you approach the decision of whether, and to what extent, you would continue to run?

Kathi Seeger
Question about consciousness, referring to the "Mindchat"-discussion you participated in in May.
I'm not going along with Philip Goff's approach of panpsychism and stating that "biology is in principle physics, therefore let's skip biology and mess around with the Core Theory" (unfairly abriged).
As soon as particels/fields make up neurons, neurotransmitters etc. from there on it may be better to talk in biological terms instead of skipping biology altogether in the discussion?

Simon Carter
Do listener numbers play a part on what topics you pick.? You have mentioned before that the Wine episode was quite low on numbers so has that stopped you from doing more episodes around this?

Sandro Stucki
In your solo episode on the multiverse you mentioned that de Sitter space is a maximum-entropy state. That seems counterintuitive: how could a highly symmetric vacuum state be high-entropy? How could there be many microstates if everything is smooth and empty?

Stephen
I have a noob question about quantum mechanics: All the strange effects like entanglement or tunnelling, how do they come about in nature? Where in nature, e.g. in the universe or in our bodies, are there ever particles isolated enough so these effects don't go away by those particles immediately interacting with something? In what situation, for instance, do two particles get entangled?
When I hear or read about quantum mechanics, I get the impression that there is always a laser involved or at least scientists working in a laboratory.

Tyler Ogorek
What are your thoughts on Pascal’s wager?

Emmet Francis
I am currently finishing up grad school and in the process of interviewing for postdoc positions in the area of computational cell biophysics. You’ve given some of your thoughts on the plight of postdocs in past AMAs, but I’m wondering if you have any advice for me to make the most of my experience as a postdoc.

Keith
In your 2019 FQXi talk on causality and the arrow of time, you briefly contrasted the nuanced causality(s) that rely on the arrow of time with the specific notion of causality in relativity where it essentially just "means that everything moves slower than the speed of light... there are light cones and all physical influences travel inside light cones." Do you mind elaborating a tiny bit here: Is a good way of putting it that the arrow of time is something that technically gets imported into relativity?

David Wych
Can deep learning methods like deep neural networks and transformers really be considered tools for scientific progress, when their underlying models of the data-generating process are not currently interpretable or meaningful? Often, they don’t even claim to be models or representations of the data-generating process.

Abhi Khune
Is it possible that there is another co-existing world made up of dark matter within our universe? Dark matter planets, dark matter life forms, dark matter spacecrafts made by those life forms etc.?

Sheldon Sillyman
I am curious about the make up of your Mindscape podcast audience. What percentage are students, science hobbyists, other scientists, other physicists, etcetera? I am hopeful that a good portion of your audience is young and excited for the potential discoveries we all can make.
[/accordion-item][/accordion]

[accordion clicktoclose="true"][accordion-item tag="p" state=closed title="Click to Show Episode Transcript"]Click above to close.

0:00:01.2 Sean Carroll: Hello, everyone, welcome to the Mindscape Podcast. I'm your host, Sean Carroll, and this is one of our regular AMA, Ask Me Anything episodes, which used to be bonuses, but due to increased responsibilities of various sorts, these are now counting as regular Monday episodes. Those responsibilities, of course, being my new job at Johns Hopkins, where I am reporting from right now, or at least I'm reporting from Baltimore, we have just moved, Jennifer and I, we're in our temporary little digs here in an apartment building, so it's not quite the smooth ready-to-go setup that I had back in LA in terms of recording the podcast. Hopefully, this will all go good, but if there are noises or whatever that are unpredictable, that's just the kind of thing that we have to deal with in life. Anyway, I don't wanna really dwell on that because I have a big fun announcement for Mindscape listeners. It's fun for me, anyway. I'm very excited.

0:00:55.0 SC: We are going to be sponsoring a scholarship for college students, or at least one student, hopefully. The idea is with the organization called Bold, B-O-L-D. Bold.org is an organization that helps people, sponsor scholarships for college students. And so they reached out and basically proposed that the Mindscape community can contribute to sending someone to college, or at least partly deferring the costs of going to college. Not all the cost, sadly. We know the cost of going to college these days are a little bit extreme, but we're looking to raise $10,000 or more. And it's gonna be easier than that sounds because I'm going to match the first $5000. What that means is if we get $5000 of other people donating, I will match up to that to make it $10,000 total. And if we go beyond that, that's great, I will stop matching 'cause I'll run out of money, but we can raise as much as we can, and $10,000 will go to a deserving student, and the next $10,000 will go to another deserving student, etcetera. It's certainly not enough to pay for college, but maybe it's enough to help someone out, and the name of the scholarship is the Mindscape Big Picture Scholarship.

0:02:07.3 SC: Of course, I've written a book called The Big Picture, but it's not about the book. It's about trying to understand the world at the biggest picture level of resolution, trying to understand the fundamental nature of reality. So the scholarship will go to people who want to do that, who want to go to college, and whether it's physics, or Philosophy, or studying the origin of life, or neuroscience, or the nature of morality, or whatever it is, people who wanna really think about these big questions. And especially, what I'm interested in doing, is helping people who might not otherwise be able to do this. So I'm looking for people who are in groups that are under-represented in these various fields, physics, philosophy, etcetera. And also first generation students. What I have in mind with this is, someone who isn't that familiar with what it means to go to college, especially in these areas, and they might think, maybe I should do something more practical, maybe I should do something that helps earn a living down the road, which is a fine thing to do, nothing wrong with that. But maybe in the back of their minds, there's someone out there who's thinking, but what I'm really impassioned about is studying these abstracts, not very practical questions about the nature of things, the beginning of the universe, what it all means, that kind of stuff.

0:03:27.9 SC: So what I hope to do with the scholarship is help them out, to give that extra little nudge that maybe can change someone's life by enabling them to pursue that thing that they're passionate about. So if you're interested in contributing, everyone is invited to contribute. You don't need even to be a Mindscape listener, but that's the primary set of targets, you go to https: //bold.org/funds/Mindscape. So that's, Bold, B-O-L-D.org/funds F-U-N-D-S/Mindscape, and you can donate money there. It's a 100% tax deductible, and 100% of it... Well, you don't get 100% of it back, would you deduct it from your taxes, but you can deduct it as a charitable deduction if you pay enough taxes to do that.

0:04:17.5 SC: What I meant to say was a 100% of your donation goes to the scholarship recipient. So Bold does not take any fraction of your donation out, they support their own administrative efforts, etcetera, through a different set of donations. You can donate to Bold to keep them going, but when you donate to the Mindscape Big Picture Scholarship, your money will be going to the recipient and Bold.org will sift through the applications to make a long list of possibilities, and then I will sift through those to decide who's gonna get it.

0:04:47.3 SC: So certainly, I hope we can get the 10K to offer one scholarship this year. It's a way that... I do make some money off the podcast, and that's good because I like money and it's great, but I do also wanna give back. This is a community, I think, of not just me and not even just the guests on the podcast, but the listeners as well, community that really feels the importance of asking these questions that are not the immediate interests of politics or culture in the world right now, but something that is a little bit longer lasting, a little bit more eternal in this nature of questions. And I wanna support people who wanna do that from a young age, and who knows what they might grow up to do, great things, maybe future Mindscape guests. We don't know. Anyway, so thanks for listening to that sales pitch. I think this is a good way for me and us to give back, for me in particular, to give back for some of the money that I do get from the podcast, doing a little bit of good for the world, hopefully. So that's bold.org/funds/Mindscape. Very, very exciting. I'll keep you updated on how that goes. And with that, we'll dive into the AMA. For those of you, remarkably, who've not been here before, this is the monthly episode where I answer questions from Patreon supporters.

0:06:00.4 SC: So the questions are asked by people who support Mindscape on Patreon. Different URL there, that's patreon.com/seanmcarroll. You can support Mindscape with a dollar per episode or whatever it is. And then if you do that, every month, you can ask questions. I don't get to all the questions. There's too many of them. I go for ones that I have interesting things to say about. Ones that are shorter. [chuckle] Ones that I haven't answered before and so on. So with that, let's go.

[music]

0:06:39.4 SC: So our first question's from Edward Morris, who says in your discussion with Ed Yong about animal sensation, I notice you never use the word qualia. Is this because you consider the whole concept of qualia, including questions like whether one organism's color spectrum might be inverted relative to another's, inherently nonsensical in a poetic natural misunderstanding of consciousness, or would the language of qualia would be something like the language of free will, a legitimate way of referring to a concept that emerges in higher level descriptions of consciousness?

0:07:16.1 SC: So you're right. Actually, I'm kind of reluctant to use the word qualia. I'm not even sure I could define it exactly, but qualia is the word that philosophers sometimes throw around to think... To characterize the experiences that we have, right? So there's the fact that you're seeing the color red, which can be explained physiologically in terms of wavelengths of light and firings of neurons, but then there's the experience of the redness of red. And that is what is supposed to be a qualia. I don't even... Is qualia only plural? I don't even know. Again, I don't use the word very much, and the reason why I don't use the word is because it seems to be a little bit slippery. I get what people are going for, but sometimes you will hear the word used in a way that makes it implied that it can't possibly be a purely physical thing. Other times you will hear the word used in a way that doesn't imply that, right?

0:08:10.9 SC: So I think that the definition or the use of the word qualia is a little bit ambiguous as to whether it is supposed to represent something that is by definition non-physical, and that's why I don't like to use it, because it's just a vague word, it just refers to different things when different people use it.

0:08:29.9 SC: So also, of course, I am a little bit skeptical about how much we know or could reasonably say about questions like, as Edward says, whether one organisms color spectrum might be inverted relative to another's. So I don't know what that means, honestly. What is inverted mean? So presumably that means something like when one person is seeing blue or one organism is seeing blue, the other is seeing red and vice versa, but I don't know what that means, 'cause I don't think there's any objective meaningfulness to saying I'm seeing red, other than the fact that I'm seeing those wavelengths of life corresponding to red, so I don't know what it could possibly mean to be... A color spectrum being inverted. So basically, these words open up some connotations that I'm not really happy with, they're not very precise, a little bit vague. So of course, if you want to use the word and define it precisely, then that's fine.

0:09:26.6 SC: But in the context of the conversation with that, I didn't really wanna take time to do that. I do believe there is an experience of seeing things, and you can give it whatever name you want. Rob Petro says, you've previously done several episodes on representative government democracy and how these things come to an end. Given the recent devastating Supreme Court rulings on the overturning Roe V. Wade, stripping states of the rights to impose reasonable gun laws, preventing judicial review of partisan gerrymandering, etcetera, what do you see is the likelihood that we are already in a failed democracy and have not just fully realized it yet? If not, what actions can we really take to address this decline apart from voting, as the persistent attacks on voting rights and fair local election rules seem targeted to minimize the efficacy of this most direct and fundamental response.

0:10:16.8 SC: So, yeah, there's two questions here, but I'm gonna group them together, 'cause this is something that we should be... Keep talking about, I think. We're not finished discussing these issues. They're very much ongoing, and they're both big picture issues like we talked about in the intro, but also of the moment politically. And that's fine, that's fair game. So no, I don't think that it's right to say that we're already in a failed democracy. What I really think is that these things are unpredictable, I think that human beings congregated together to make societies, to make democracies, are hard to predict exactly what's going to happen.

0:10:53.8 SC: And then this is part of the idea behind the physics of democracy project. It's because unlike in Isaac Asimov's psychohistory in his Foundation series, where he said if you get enough people together, their idiosyncrasies will average out, and you can exactly predict what's going to happen. I think he was wrong about that, because you do get a bunch of people together, and there is a collective behaviour, but even the individual people are complex systems. It's not the same as getting atoms and molecules together to make a fluid or a gas. In that case, the dynamics of the atoms and molecules are sufficiently simple that you can average over everything and get more or less predictable deterministic behavior on the macroscale. But when your collective phenomenon comes from a bunch of people who are themselves pretty darn complex, then that averaging just doesn't work. And so what you can do instead is talk about the probabilities, the possibilities, I guess, as well as the probabilities. Is it possible that we are... That our democracy is failing? I don't think it's failed. And again, let's pause to point out that people just want to be cheaply cynical and say, "Well, we're not a democracy, 'cause some people have more power than others, or my vote doesn't count, or whatever."

0:12:14.8 SC: Fine. Okay, that's a point you're trying to make. It's not the point we're talking about right now. What we're talking about, the real difference between the ability to vote at all, the ability to have elections where there are multiple parties and you're not sure who's gonna win versus systems where it's clear who's gonna win all the time, 'cause there is only one party, or there's no parties at all, there's just an autocratic ruler, okay? That's the distinction we're talking about right here. Right now, by those lights, we are a democracy, okay? And again, there's another bunch of people who wanna be nit-picky about a republic versus a democracy. We are a Democratic Republic, which is both a Republican and a democracy. That is the correct way of thinking about it. So anyway, those caveats aside, we're a democracy right now, but there's no reason to think that we will be forever. Democracies fail. The Roman Republic lasted for 500 years, which is hugely long.

0:13:07.1 SC: Right now, it depends on how you count, but by some ways of counting, the United States with its constitution is the longest, persistent existing Democratic Republic in the world. There's some footnotes there also because, depending on how you count, there's some subsets of countries that are ruled over other countries, but they vote, etcetera, etcetera, but big established clear-cut countries that have been having a democratic system, the United States is the longest existing one right now.

0:13:41.0 SC: And it's not that long, right? Two and half centuries may be, less than that if you are just counting the constitution. But it's still pretty long compared to a generation. Right? Compared to the life span of a person. So there's no rule that says we have to last for another 2 1/2 centuries. We could last for another 2 1/2 years, we just don't know. And that's not to say, because we don't know anything goes, we can certainly pinpoint probabilities, we could have credences that the democracy that we have right now is going to last, and it won't. And I don't like to be an alarmist about these things because I think that people like to get attention by being alarmist and talking about all the dangers, but this is a danger, I think is super-duper real, and this is one of the reasons why, on the podcast, I try to talk about it every so often, even though it's not one of the top two topics that we talk about here, I do want to keep reminding people about this fragility, we're a complex system, there can be bifurcation, phase transitions, there can be sudden shifts caused by tiny changes in the individual states of every person. If every single person moves a little bit in an anti-democratic direction by just a little bit, that has a huge impact on the likelihood the democracy will survive.

0:15:00.5 SC: And I think that there are many complications here, I've talked directly about democracy, but also about polarization with people like Will Wilkinson and Ezra Klein. We talked about democracy with people like Astro Taylor and Stasavage and Edward Watts, who talked about the decline of the Roman Republic. And it's a complicated thing because it's not just you vote and the majority wins. That's not the system we're in, and that would be a bad system, I don't think the direct democracy is very fruitful to explore in this modern world, because things are too complicated. You need representatives, who, even if they themselves are not experts, their job is to talk to the experts and figure out what to do. I think that's a better system than just letting everybody vote on everything. But what it means is that there's all sorts of mechanisms by which a minority of people can basically take over the country. The Supreme Court, the electoral college, the Senate of the United States Congress, these are all institutions which wield an enormous amount of power and are something other than let's just let the majority have its way.

0:16:12.4 SC: And that's... There's a reason for that. The people who wrote the Constitution in the first place did it for a reason. They wanted to protect the rights of minorities, but there's two things that are happening here. It's good to protect the rights of minorities, but number one, the differences in representation, difference in population in different states of the US now is enormously larger than it used to be. So the slight malapportionment that you get by giving each the same number of senators wasn't a big problem back in the 1790s, it's a much bigger issue now. And the second problem is that you have to distinguish between protecting minority rights and allowing minority rule. So the obvious instability of a setup like ours is if the mechanisms that have been put in place to make sure the minority has a voice, are hijacked by the minority to take power and keep it. If you can put mechanisms in place that prevent a majority from electing the people they're trying to elect, then that can be a self-sustaining problem for your purported democracy.

0:17:21.7 SC: And I think this is... You don't need to be very aware of current events to know this is a very, very real problem, a very real prospect, and this is going to happen because it's not a secret. [chuckle]

0:17:34.2 SC: You see the plans being laid out in front of us. They tried in the 2020 election. They're gonna try again in 2022, 2024, etcetera. And not every Republican wants to do this, but plenty do. You can look at the polls, and you can look at the plans that are just being discussed right out there in the open. You can look at what the states themselves are doing. Basically, for those of you who are not Americans, the issue is that we have an electoral college, so states, the 50 states of the US, elect a certain number of people who then vote to see who's president. And roughly speaking, that is almost supposed to be representative because unlike the US Senate, where every state gets two senators. In the electoral college, bigger states get more electors. The reason why it's not completely representative is because usually the states allocate their electoral college members, winner-take-all. So a big state that just goes 51% for one person and 49% for the other, all of their electoral votes go for one person. So it's a kind of mixed up system for that reason. But again, there were reasons to do it. The problem is you're supposed to...

0:18:47.9 SC: In the modern systems, this is not the original system, but in the modern system, you're supposed to just allocate those electors on the basis of the people who voted in your state. And what's happening right now is that states are putting in systems which let the state legislature over-run, over-rule, the vote of the actual people, so you could have a big state which voted for one person rather than the other, and the state legislature just says, "No, we're not gonna send those electors to the electoral college." This is the explicit plan being put in place right now. And the cover for it would be, well, we don't believe the vote totals because we think you're cheating. There's been essentially zero evidence that anyone is cheating, there's voter fraud or anything like that, but that is the rhetoric that is used to justify throwing out the actual results of the election. And we're in a polarized, pretty evenly split country, so a tiny amount of this can go a long way. It's not completely absurd to think that one person could win the popular and electoral vote by a lot in 2024, and yet shenanigans at the state level could end up with the other person being elected president, and that person then gets to pick more Supreme Court Justices and more local officials will come into power because of this, and so there is a positive feedback mechanism that pushes us away from truly democratic representation.

0:20:15.0 SC: This is a real, huge, enormous problem. And the other question that Rob asks is, what we do about it? Yeah, I wish I knew. I think there's another question down here that is related to that, and of course, there's the simplistic thing, vote. [chuckle] Get your friends to vote, whatever. But exactly because of the system that we have, the people who are most worried about the dangers to democracy or those whose votes don't matter that much, because they're probably living in states where the election is very one-sided for one side or the other, right? It's a small number of states that can possibly have this crucial impact, and this is why, by the way... I know I'm rambling on about this, but it's just really, really important, and I don't... I feel a little bit powerless, I don't know what to do about it, but there's of course been a push against the Electoral College for exactly this reason, because it's been several times in the past few elections that the person with fewer popular votes has won the electoral college. And because the United States is more or less, at the current moment, a law-abiding institutional government, we have given the election to the person who has won fewer electoral votes, George W. Bush, then Donald Trump.

0:21:33.6 SC: We listen to the rules, okay. And so... But the rules are dumb. The rules don't work anymore. They don't apply to... The conditions that originally made people want to put these rules in place, no longer apply. So we should be protecting minority rights, the rights of states that have smaller numbers of voters, etcetera, but we can do it in a much better way than this. And so some people are saying, Look, just get rid of the electoral college. And I think that makes perfect sense, honestly. There's no reason why the President should be elected by this weird Rube Goldberg system, and there's a very dumb counterargument that says, "Oh, if you get rid of the electoral college, then small states will be ignored because you'll just wanna win the most votes in the popular votes. You'll spend all your time in New York and California and Texas, and you'll ignore all the small states." That is an entirely bogus argument because you're comparing it to the current system with the electoral college. And in the current system, any state, whether it's Republican or Democrat, bigger, small, urban or rural, any state where it's more or less clear which candidate is gonna win, gets completely ignored in the election.

0:22:46.0 SC: So no one is out there in the presidential election campaigning in Wyoming and Montana. They are small states, population-wise. They're large in terms of square footage, but no one's gonna campaign there because they're gonna vote for the Republican every time, so why bother campaigning there.

0:22:58.9 SC: And also why bother campaigning in New York or California where they're gonna go democratic almost every time. That's a recent phenomenon, but in the current situation, that's what's gonna happen. In a direct popular vote for the president, there'd be equal reasons to campaign everywhere, up to the fact that you can sort of reach more people in a big city. But these days, most of the campaigning is not done by literal rallies, that's not how you reach most of the people. You have ad buys and commercials and so forth, and those who would go into places they don't go now if you got rid of the electoral college. So I think there's no reason to keep the electoral college right now, but there's also no institutional will to get rid of it. And the one... So anyway, what can you do about it?

0:23:47.0 SC: You can vote and you really can vote. There's other people are saying, "Well, we've been voting and it hasn't worked." Well, vote more, have other people vote more, that's the most obvious direct thing you can do. Number two, you can help persuade other people, right? And this is something where I'm not an expert, but there are experts out there, how do you persuade other people, how do you actually get people to your side and you don't go... You don't immediately go for the people who are way far on the other side. That is a lost cause on... People on either side of any highly polarized debate are not the persuade ones, but there's always huge numbers of people in the middle. Those are the ones you should be thinking about reaching, and so how do you reach them? It's not haranguing them and telling them they're stupid, alright?

0:24:28.1 SC: That's not how to do it. You have to be persuasive, and you have to be friendly, and you have to present the case that people like you are probably on the right side, okay? That's a difficult thing to do. There are techniques that have been developed for doing that.

0:24:43.0 SC: Study up on those and try to learn them. And the other is, even if you live in a state which is very much on one side or the other, you can still maybe influence your actual representatives in Congress or the state legislature or whatever by asking them to do something. I will vote for Democrats compared to Republicans right now. When I was younger, I would vote either way, 'cause I don't actually want to be starkly ideological about these things. But the modern US Republican party is just an embarrassing disaster, and I cannot imagine voting for it right now. Most of the people don't even believe that the election was fair, which is a complete crazy nonsense. So it's clear that I'm voting Democrat in the immediate future, but what you can be legitimately disappointed about is that our Democratic representatives don't take this problem nearly seriously enough. See, they're not really nearly as worried about the decline of democracy as they should be, they sometimes give it lip service, but if you've been in this business a long time, there's a tendency to treat everything as business as usual.

0:25:49.0 SC: And that's a huge problem 'cause this is not business as usual. This is not typical partisan infighting. This is not just everyday politics. This is an existential threat to our system of government, and because it is so unique and so dramatic, it calls for unique and dramatic responses. And we've not been getting those ever from the people nominally on our side, for those of us who are Democrats. So even if you're in a deep blue state, etcetera, you can still try to put pressure on the people who are Democratic representatives to say like, "Look, take this problem seriously." Do it by writing to them, by emailing them, by visiting them.

0:26:27.8 SC: It's actually easier than you think to visit Washington DC or your local congressional office and at least talk to a staffer, if not to talk to a person, the actual representative or senator or whatever, but sometimes you can even do that. So try to do all those things. I don't know if it's gonna help, but taking the problem seriously and really thinking about what to do about it is the major first step, which a lot of people don't even do as much as they should. Anyway, sorry for that very long thing, but I could go on. Maybe there should be a solo episode, but I think I said most of the important things I wanna say.

0:27:05.2 SC: Peter Blankenheim says... Just to change topics here a little bit, the BABAR group at the Stanford Linear Accelerator recently observed that the B meson oscillation to his anti-particle takes less time in the reverse oscillation, violating T-symmetry, T for time. When that got called an arrow of time, you objected, saying that the arrow of time comes from the low entropy of the Big Bang and said indeed the entire phenomenon of T violation has absolutely nothing to do with the arrow of time. But wait, a reaction that is faster than its reverse is pretty special. Why doesn't that qualify as an arrow of time?

0:27:38.4 SC: Well, it's pretty special, and it's certainly very interesting. So I don't know how recently this was. In my recollection, the thing you're referring to was a few years ago, but yeah, we know that there is a symmetry or at least a transformation in particle physics, in quantum field theory, called T, capital T, time reversal symmetry, okay. And we know that this is violated in the standard model of particle physics, we've known that for a long time. We sort of known it indirectly, and the recent experiments have been seeing it directly, okay? So that's the improvement, but no one is surprised about this recent finding, it's just a little bit more direct than we had before, but it's not narrow of time, and the reason why is because it's perfectly reversible.

0:28:18.2 SC: And so this is a subtlety. And actually, I will talk about this in the book coming out in September, The Biggest Ideas In The Universe volume one, because this issue of reverse ability versus an arrow of time is a very subtle, but an important one. The thing about the arrow of time, what we usually call the arrow of time, again, you're welcome to use the phrase arrow of time however you want, it's a free country, but what we mean generally in physics and philosophy by the arrow of time is an irreversibility of things. It's not that there's a different rate of things going one direction or another, but things only go in one direction, entropy increases. And of course, at a very small scale where entropy is not very well defined, it can fluctuate up and down, but in the macro world, ice cubes melt in warm glasses of water, but glasses of water not spontaneously form ice cubes and hot water in them, okay.

0:29:12.0 SC: And the point about T-violation is that it is a little bit of a definitional issue going on here, it's a little bit less solid than you might think from usual particle physics discussions. What do you mean by T-violation? So you say, Peter, in the question, a reaction that is faster than its reverse is pretty special. Sure. But what do you mean by the reverse, okay. So that T-symmetry or that T-transformation has to be defined very specifically. What gets reversed? Think about classical mechanics. Okay.

0:29:47.4 SC: Think about a system of objects, particles with positions and momenta, and you say, okay, that's... It's just perfectly classical. It's perfectly time reversal symmetric, right?" We all agree. But what do you mean by time reversal, you take the system at its initial state, you evolve it forward in time, and then you reverse it backwards, and it comes to back to where it is. But what do you mean by that reversing backwards? So the answer is, you need to both take the... You need to take T to minus T, okay? You need to take time to minus time, so you literally run it backwards. But also if you've taken the system, evolved it forward, before you go T to minus T to reverse it, you need to flip the sign of all the momenta of the particles, okay? Every particle has a velocity, mass times velocity is momentum, and you have to reverse the direction of that momentum, because otherwise it's gonna just keep moving in the same direction it went in, and it's not gonna reverse at all. And this seems very innocent, I flip the momentum to minus the momentum. What else could I do, really? Momentum is mass times velocity.

0:30:50.4 SC: Velocity is derivative in respect to time, it's gonna flip. But it's a little bit of a important move there, because when you get to other kinds of systems, like systems in quantum field theory, there's no momentum variable, there's a wave function. And so what exactly do you do to the wave function to flip it, to reverse it, and then evolve it backward in time. And so particle physicists, quantum field theories have invented a procedure that flips the sign, the time reverses the sign of the wave function, so you can evolve it backwards, but there's a little bit of arbitrariness involved in that procedure. And so in particular, in particle physics, there are also transformations called C and P.

0:31:33.5 SC: Charge conjugation, which is flipping particles with anti-particles, and parity, which flips space as well as time. And we know that even though T is violated and C is violated and P has violated, the combination, CPT, is preserved. So if you take a system, evolved it... Quantum mechanical system, evolve it forward in time, and then for a little bit, and then you stop it and you reverse not only the time direction, but also the space directions and the particles to anti-particles and then run it backwards, you get to exactly where you started, okay? That's CPT symmetry.

0:32:14.1 SC: But here's the thing, I could just define what I mean by time reversal as what most people call CPT. No one can stop me from doing that. And you can... Again, you're free to define it however you want, but there are... The important, non-trivial physical fact is that there is a time reversal operation that is perfectly preserved by the laws of nature, and we just happen to choose to call that CPT rather than calling it T. But it is a reflection of the underlying reversibly of the fundamental laws, and that is why it is not an arrow of time, because every place that you go to from an initial starting position can be reversed exactly to where you started without any loss of information or ambiguity. You can't do that in the macroscopic world. If you see a glass of water, you don't know whether 10 minutes ago, it had an ice cube in it that melted or not, it is irreversible.

0:33:10.9 SC: So that's the important physical difference between these T-symmetry violations in particle physics and a real honest-to-goodness arrow of time. Alright, I'm gonna group two questions together. One is from Lima, I think is the pronunciation, who says, I've been wanting to ask for a while what your actual take on Stephen Wolfram hypergraph project is." And Brandon Lewis says, I see a lot of overlap between how you describe quantum gravity in your early episode on the topic, and Stephen Wolfram's graph theoretical approach to fundamental physics. I was left with the impression that you don't find Wolfram approach all that compelling, could you expand a bit more on this topic? So yeah, so both are asking what I think about Stephen Wolfram's project.

0:33:49.2 SC: I'm not being coy, I'm not trying to hide my opinion about this. I think that I said it in the intro to that episode when I had Stephen on, and so two things are my reaction to it, one is, I'm very glad he's doing it, I'm very glad that people are trying out of the box unconventional approaches to fundamental physics. I do think that there is a bit too much conventionality in how we approach these very difficult problems, I think that we need more variety in our approaches. And that's easy to say, but in practice, what everyone means when they say, yes, we need more variety in our approach is, we need more approval for my approach. We need more support for what I'm trying to do. It's very hard to support research for an approach in fundamental physics that you personally don't approve of, you don't think it's very likely to succeed, but you are gonna prove doing it just because we should be doing different approaches. So I really think we should try to do that, anyway, and that's what I try to do. So I don't think that Wolfram's project has a great chance of success, but I do think that it's worth supporting for that reason, why...

0:35:01.3 SC: The second thing is, Why don't I think it has a great chance of success, I think that there's basically two possibilities in what I'm doing and what other people are doing, in trying to understand fundamental physics more deeply, is trying to understand quantum mechanics more deeply. Quantum mechanics is a framework, not just a specific theory, so all of the successful theories of nature that we know of right now are within this quantum mechanical framework. We have zero evidence or intellectual reason to move beyond quantum mechanics. We should still think about what that would mean to do so, just so we can test it etcetera, but there's no evidence, either empirically or even theoretically that says that quantum mechanics is wrong, okay? And furthermore, I would argue that we haven't tried nearly as hard as we could to work out what fundamental physics should be like using conventional quantum mechanics. That's what I and my collaborators are trying to do. So I think that when you have a program that starts from something other than conventional quantum mechanics, which is what Wolfram's project does, there are two possibilities. One is what he will find is that they just rediscover quantum mechanics. [chuckle]

0:36:15.7 SC: In which case, great. But I already knew quantum mechanics, and therefore I'm working on that directly, using that hint that nature has given us that this is the way things work. Or, alternatively, they will not discover quantum mechanics, they will discover that their theory is different than quantum mechanics, which would be great, but then probably it's not gonna work, 'cause quantum mechanics is super-duper successful.

0:36:39.1 SC: Of course, there's a third option that it's different than quantum mechanics, and it does work, and that would be hugely important and really, really interesting, so that's why they should keep trying to do it, but I'm not gonna put a lot of money on doing that, and the other reason why putting money on it seems ill-advised to me is that it's hard to make progress in fundamental physics. It's hard to really come up with an important insight about how nature works at the most fundamental level. And historically, the way that we've made really, really important leaps of understanding is that we've been nudged in the right direction by data, right? By empirical information that the world has given us.

0:37:21.7 SC: I forget who originated the idea, but Sidney Coleman, my old quantum field theory professor certainly liked to say that hundreds of philosophers could not have gotten together and invented quantum mechanics before the 20th century. Classical mechanics were just working very well, and no one would have had the idea of coming up with quantum mechanics until the data forced us into it. So even though it might turn out to be the case that we need bigger, bolder ideas in fundamental physics, the idea that we're just gonna guess the right one and then work it out and it's all gonna work, just seems very unlikely to me. It could happen, so that's why I wanted it to be explored, but I'm not gonna spend my time exploring it 'cause my life is too short. [chuckle]

0:38:05.9 SC: I need to work on things that I think are a little bit more likely to eventually succeed. Okay, Paul Nurse says, "Thank you for the solo episode, number 200 on the philosophy of the multiverse. In that episode, you mentioned that if the cosmological constant stays constant, the universe would expand into something approaching the De Sitter Space. You then describe this as a nearly perfect black body with a defined temperature that last for an infinite time. Why should the universe ever stop expanding or cooling, why would it remain in any stable state with any defined temperature instead of continually expanding and cooling?

0:38:41.6 SC: Good, this is very straightforward question about the quantum nature of De Sitter Space, which is fine. I probably just glossed over that up a little bit too quickly in the solo episode. And because it is a tricky, interesting, subtle feature of the interplay of general relativity or classical theory of space-time and quantum field theory, even if we don't have a full theory of quantum gravity yet, we do know a lot about what we think of as quantum field theory in curved space-time. So the idea... The framework in which we treat space-time is purely classical, and then we do quantum field theory on top of that, okay. Quantum field theory in curve space-time. So we're not quantizing the gravitational field itself, we're quantizing everything else. And this is not supposed to be a good theory of everything, it's suppose to an approximation to what the full correct theory of quantum space-time is actually like.

0:39:36.0 SC: In that approach, you can ask about quantum fields in De Sitter Space. And again, general relativity itself, even classical general relativity without any quantum fields, there's a lot of subtleties about words like expanding, okay? Is De Sitter Space expanding or not? So De Sitter Space was the very first... Sorry, that's not actually true. The history is that Einstein invented cosmology, modern cosmology, by imagining the universe was completely smooth in every direction, plugging it to his equations and finding that the universe had to either expand or contract, and he didn't think that that's what the universe was doing, there was no empirical evidence that the universe is expanding or contracting, so he put in the cosmological constant to balance it and keep it stationary.

0:40:23.1 SC: And then what De Sitter did, Willem de Sitter, Dutch astrophysicist was to say, okay, what Einstein did was put in matter plus the cosmological constant and balance them. What if we just removed the cosmological constant... Sorry, I got it exactly wrong. What if we just remove the matter, have nothing but the cosmological constant, and he found an exact solution to Einstein's equation. And has happened many times in the history of GR, Einstein himself was perturbed by this. He didn't believe it, he didn't like it, but he eventually came to De Sitter side and he and De Sitter actually worked together to invent another solution. But anyway, the De Sitter solution has nothing but cosmological constant.

0:41:00.7 SC: So it turns out, as a subtlety of general relativity, that you can take this space-time, four dimensional De Sitter Space, nothing in it, but vacuum energy, but the cosmological constant, and you can slice it in different ways. By slice, we mean it's a space-time, you can think of it as space plus time separately. So the slicing is, slicing the space-time into moments of constant time that we call space at any one moment in time. In good, old ordinary flat space-time, there's more or less an obvious way to do that, and we all agree what space and time look like. In De Sitter Space, it's not obvious, there are different ways to slice it. Some ways of slicing De Sitter Space make it look like it's expanding, other ways of slicing De Sitter make it look like it's not expanding, like it is, in fact, stationary.

0:41:52.0 SC: It's a complicated thing. It's a subtle thing, and you really gotta dig into the details of the mathematics of general relativity to understand what is going on there, but the point is, I'm just pointing the ambiguity of thinking of De Sitter Space as expanding. When you have matter in the universe as well as the cosmological constant, then there's an obvious natural way to slice the universe into space and time, namely slice it so that on each slice, the density of matter is more or less uniform, so over time, the density of matter will go down in an expanding universe, it will increase in a collapsing universe, but across space at any one moment, the uniform... The density is more or less uniform. So then you can say, Well, what I mean by expanding is in those slices, the density is going down, the space is expanding. But when there's no matter around, when you have nothing but the vacuum energy, there's no right or wrong way to slice it, so you can't even say whether it's expanding or not. Anyway, that was a slight digression to get the subtleties of De Sitter Space exactly right. But if you say, Well, okay, but in this case, we're not exactly in De Sitter Space, we're in our current universe which will eventually empty out more and more, but there's always gonna be like one or two atoms floating around, so slice it in that frame that is given to us by that thin gruel of remaining atoms, okay?

0:43:10.9 SC: So the universe does not stop expanding, to answer Paul's question, but it does stop cooling, and the reason why is because the temperature is no longer a feature of the physical matter, and again, this is a subtlety, sorry about this, but no one ever said general relativity and quantum field theory would be easy. In the current universe, we have a cosmic microwave background. We have photons out there in space with an average temperature of about 2.7 Kelvin, pretty darn cold. But in De Sitter Space, it's not those photons that are giving us the temperature, it's the temperature of the vacuum itself, it is a feature of De Sitter Space that empty space, the lowest energy state you can possibly be in as a quantum state for the quantum fields that has a non-zero temperature, and the temperature is very, very low, is 10 to the minus... I don't know what it is, 10 to the minus 30 Kelvin, something like that.

0:44:12.6 SC: Eventually, it will be way, way lower than the current cosmic microwave background is. See, I've been doing philosophy for too long, I've forgotten all the numbers that I used to know for the universe. But this is a feature of the vacuum state in a space-time that has a horizon, it's exactly the same kind of phenomenon that gives black holes their temperature. A black hole has a temperature, even if there's no stuff around it, it's because the vacuum state of the quantum fields in the presence of the black hole's event horizon is a thermal state, one with a non-zero temperature. In the case of De Sitter Space, there's a horizon, not over there where the black hole is, 'cause there's no black holes, but around you in every direction, there is a horizon surrounding every observer, and that horizon has the feature that it gives the vacuum state of the quantum field theory a non-zero temperature. I'm not gonna go into more details than that, but the point is, roughly, if I wanted to cut this short, to answer Paul's question, in De Sitter Space, in the way that you're thinking about it, the universe is still expanding, but it is no longer cooling.

0:45:19.8 SC: It approaches a state with a constant expansion rate and a constant non-zero temperature, and if you're worried where the energy comes from, it's from space-time itself. That's the best we can do. Alright, Tom Hawkins says... I better hurry up, I'm getting... This is giving too much time to each question here. Tom Hawkins says black holes have been proposed to be the source of some, if not all of the dark matter. Do you have any comments on this? You know, look, it's possible. So the reason I wanted to answer this question, remember, for those of you who don't already know, I choose questions on the basis of not if the questions are good or bad, but on, do I have anything interesting to say about them that might not have already been said. In this case, the thing to emphasize is black holes are perfectly good candidates for dark matter. What do you want for dark matter, you want it to be dark, so no electrically charged particles, that's a good idea. And you want it to be matter. In other words, not radiations, you want them to be slowly moving particles, generally that means massive, but at least a little bit of mass. They can't be moving at the speed of light.

0:46:21.1 SC: And you want them to be distributed in more or less the right way, and that's a more technical requirement, but there you go. Other than that, anything that fits those categories, those characteristics could be dark matter, so the most popular particle candidates are Weakly Interacting Massive Particles, WIMPs and axions, which were proposed in the '70s to solve a problem in the strong interactions of the standard model. Black holes are another perfectly good possibility, they could be small, but they can't be too small. If the black holes are small, they evaporate away, and they're no longer dark matter. But if they're big enough to not get evaporated, then they could be massive particle-like things that are cold, cold, dark matter. The problem is, the reason why they're not nearly as popular as these particles... Well, let me say, the good thing about black holes is they're known to exist, okay, so unlike the WIMPs or the axions, we know that there are black holes. We've seen them via LIGO and the Nobel Prize was just given to people who study the black hole, the center of our galaxy and Sagittarius A.

0:47:28.1 SC: The bad news is we don't know a way of making them in the right abundance to be dark matter. And that's the crucial thing, which I think that people don't always appreciate, that for the WIMPs and for the axions, it's not just that they're are particles that are dark and matter-like. That's not why they are the popular dark matter. Again, it's just part of it. But the real reason is that the evolution of those particles in the early universe can very naturally give rise to exactly the right amount of particles to be the observed amount of dark matter. You don't have to work very hard. In the case of the WIMPs, it happens almost automatically. This is what we call the WIMP miracle. If you have Weakly Interacting Massive Particles, you don't have to try very hard to get exactly the observed density of matter. In the case of axions, it's not as clear. You can get the right abundance, but you have to like tune a parameter. It's not like tuned to be some special value, but you have to fix an arbitrary free parameter to get the right abundance of particles.

0:48:32.3 SC: And you can do that, and it's not that hard to do, and there's no reason not to do it. So you can think of it as there was a parameter we didn't know, and we've observed it by measuring the abundance of dark matter. But in either case, it's not that hard to get the abundance. In the case of black holes, where do these black holes come from? Well, who made them? You have to work pretty hard to invent a mechanism in the early universe that makes that many black holes, 'cause remember, the amount of dark matter is something like five times as much as the amount of ordinary matter in the universe. So where did all that density of black holes come from, and you can do it, you can sort of play games with very, very large density fluctuations at very, very small scales in the early universe that are so small, you don't see them directly the microwave background or whatever, but big enough that they sort of collapse into black holes, but it's hard. It's not natural.

0:49:24.8 SC: It's very baroque, very awkward-looking. And so, black holes are absolutely a candidate, but they're not a very popular candidate in the list of most working physicists. Andrew Goldstein says, respect for data generated by and conclusions drawn from scientifically sound research appears to be in decline, especially when it contradicts held beliefs. How do you think the rest of us science believers can help facilitate more people to listen to what science has to say about everything from vaccines to climate change to emergence. Emergence is a bit of a surprise there. I didn't know emergence was like a hot topic in science denialist circles, but yeah maybe it is. Maybe I've not been reading the right Twitter feeds. Yeah, and this is gonna be a similar... I wanted to answer the question because I think it is an important question to ask, but I don't think that my answer is gonna be especially enlightening. Similarly to the question about how we support democracy, the short answer is, I have no great ideas here. Again, there are people who have studied this, and I read a little bit of what they've written. There's little tricks, like if you want to dissuade people from believing some myth, okay, so let's take the earth is flat. This is a special case, so it's not a great example, but at least most people listening to this podcast are not gonna believe the Earth is flat, so I'm not insulting anybody.

0:50:46.4 SC: It turns out the studies have shown, and these are difficult psychology studies, so you have to worry if they're replicable and so forth, but people try. If you want to persuade people the earth is not flat, you do not do things like say, Some people say the earth is flat, here's why they're wrong. And that's a bad strategy for two reasons. Number one is you're leading with the myth, with the mistake, and very often, apparently, this is what the study seem to say, when you do that, the thing that people remember is the myth, not the refutation of the myth. So you have to lead with the true thing first, and then you can say, as opposed to the myth which says this. Or you should say, the earth is round, despite some people thinking that it's flat, etcetera. The other thing wrong with it is that it's just argumentative rather than persuasive. As I mentioned in the democracy discussion, you have to treat the people you're talking to like people, like people who, like you, are trying their best to understand things and get the right answer, and making mistakes sometimes.

0:51:53.9 SC: We all make mistakes sometimes. Now, of course, again, there are extremists, there are people who are not trying to get the right answer, there are people who are just lying, for whatever reasons, forget about those people, okay? That's a different problem. I'm thinking about the vast number of people who are kind of trying their best, but maybe they're listening to bad resources or just have a reason that a certain problem is difficult for them, or they have some prejudices of their own, etcetera, which basically means all of us. We all have biases, we all are making mistakes somehow, and it's very, very hard to realize that we've been wrong. So rather than just arguing with people who don't wanna be argued with, try to see things their way, try to legitimately ask yourself, why would they think that?

0:52:40.4 SC: If someone believes that vaccines cause autism or cancer or whatever, or if someone believes the Earth is flat, or climate change is just a hoax, hooked up by the federal government and research scientists to get more grant money, try to, in good faith, yourself, figure out why they would believe that. If you're talking to someone one-on-one or in a small group be... And don't fake it, really try to understand why they would think that, and that means both listening to the reasons they would give for thinking that, but also maybe digging in a little bit more deeply and wondering why they might be so distrustful of authority, maybe they have some very good reasons to be distrustful of authority, and that leads over into their beliefs about other things.

0:53:26.3 SC: And then work within their existing belief systems, say like, Well, if you think this, then maybe you should think that, okay? Try to discuss, try to have a legitimate conversation rather than just lecturing and hectoring people. Now, in the broader picture, when you don't have a one-on-one discussion, but you're just broadcasting out there into the world, again, part of it is just saying true things and letting those true things propagate. Another is to say true things in a way that seems friendly and approachable and reasonable and adorable. Be the role model, be the good guys, be the people that other people will look up to and want to be, rather than being an annoying scold. And I try to do this myself, sometimes the temptation to being annoying and scold is just overwhelming and I give in, we all do. But this is how we can try, and none of this is guaranteed to work, I guess, so I don't know. So that's why I'm saying I don't have great suggestions here, but I do think the problem is important, so I want to let people think about it themselves, maybe the people listening will have some better ideas than I do.

0:54:35.8 SC: But again, if you think you have better ideas, that's great. I'm happy to hear them myself and leave them in the comments section, etcetera. But what I would ask is have evidence-based ideas. Don't just say, Well, I think this is how you do it. Okay, look for evidence that this particular strategy that you're suggesting actually works. Those are the suggestions that are really interesting. We all have impressions, but these are difficult questions, how to talk to other people and how to change their minds, and so ways that are backed up by science are the best ways to think about. Okay, Anonymous says, I saw a PBS space-time video where Matt O'Dowd explores whether it's possible that the Big Bang is a time-reversed black hole, aka a white hole. My question is supposing that it's possible for a white hole to have black holes inside of its event horizon, would it also be possible for a black hole to have white holes inside of its event horizon? So I think there are some things that I can say with confidence about this and other things that are a little bit trickier.

0:55:38.2 SC: I mean the short answer is yes. It absolutely would be possible because the fundamental thing to keep in mind is what is a white hole? A white hole is exactly a black hole, played backward in time. This is why we don't see white holes around us all the time. A white hole would be something out of which things could come, but you couldn't get into. Everything about a white hole is just come up with the analogous thing for a black hole and say it backwards in time. So a black hole is a region of space-time you can enter but never leave, a white hole is a region of space-time things can come out of but not go into. Okay, that's what it is. So a black hole has a singularity in the future. If you enter a black hole, you will hit the singularity inevitably. A white hole has a singularity in the past, the singularity out of which things come. You can't get there as you are traveling into the future. So the reason why there's no white holes lying around is that they are thermodynamically crazy. It's exactly like having scrambled eggs turning into eggs or glasses of water spontaneously forming ice cubes, etcetera, all of these reversing entropy things are analogous to white holes.

0:56:45.8 SC: White holes decrease the entropy of the universe just by existing, okay? Now, the one big exception to that is the Big Bang itself, because we have this rule. The Second Law of Thermodynamics. Entropy increases with time. Why does that happen? Well, a big part of the explanation is we started out at early times with very low entropy, and that's the part we don't know why that happens. Some of us have ideas, but we're not sure why. So the Big Bang cosmology is highly analogous to a white hole, it really is very, very similar. It's not exactly, but it only depends on exactly how you're defining white hole, because sometimes for a black hole, we define it as a region of space-time that is finite, that is somehow bounded in size, I mean the black hole is not the whole universe, it's just part of the universe, and you can enter it. The very idea that you can enter the horizon but not escape implies the horizon has a finite domain there, right, that's the real only difference between the Big Bang cosmology and a white hole, because a white hole has a singularity in the past, the Big Bang has a singularity in the past, the white hole starts with low entropy, and the white hole would do that...

0:57:57.9 SC: I'm not even sure, I guess you could do it either way, but traditionally, you could do it if you were... I don't know what to say about that. I think a white hole could either have a low entropy or high entropy, but the actual Big Bang starts with low entropy and entropy increases from there, but there's a thermodynamic miracle that is involved, namely why the original Big Bang had such a low entropy at those early times. And of course inside our white hole-like cosmology, there are black holes formed, so for the question, could you have black holes inside white holes? Just time reverse everything, and you can get everything you want. That doesn't mean it's thermodynamically sensible, but there will always be some solution to the microscopic equations in which that happens. I guess this is relatedly, so Lewis B says, I'm curious about the past hypothesis's implications on our ability to rigorously study the past hypothesis. You seem to indicate at times that even when trying to scientifically study the entropy of the early universe, the mere process of making observations and drawing inferences itself must already presume the past hypothesis. It would seem that we are forced into at least some form of circular reasoning while trying to conduct science on the question of the entropy of the early universe.

0:59:15.2 SC: So I know what you mean, Lewis, but I don't think it's circular reasoning, really, I would say it's a matter of being self-consistent. Okay. So what is being referred to here is that we observe things like the distribution of galaxies at early times or the cosmic microwave background radiation, and we say, Oh look, it's in a configuration which has lower entropy than the universe today, and that's not very hard. Most of the entropy in the universe today is in the form of black holes, and we don't see any black holes in the past, so it's the conclusion that the entropy of the early universe was lower at early times does not rely on any fancy observations we have to do, just that there are a lot of black holes now and there weren't that many then, big super massive kinds of black holes. The entropy of the single black hole at the center of our galaxy is larger than the entropy that the whole universe used to have near the cosmic microwave background era. Okay, but how do we get to that conclusion? Because after all, as I've said in other contexts, when we observe the microwave background or distant galaxies or whatever, we're not observing them from the point of view of when that light got admitted from those phenomena.

1:00:26.6 SC: In the story that we tell, the cosmic microwave background is light that last scattered about 380,000 years after the Big Bang, and now the universe is like 14 billion years old. So we are inferring what things were like on the basis of the light that we're observing today. What we're actually observing is not conditions 380,000 years after the Big Bang, what we're observing is a radiation coming to our telescopes today, and we use our theoretical framework to work backwards to see what that implies about the early universe at early times. And we say it implies it had a low entropy, but that does imply... That does assume that entropy has been increasing all along. So you assume that entropy has been increasing, and then you infer from your observations that the specific kind of low entropy state looks like the following thing. So we're not assuming the specific low entropy early state, we're assuming the basic rough idea that entropy is increasing, then taking the observations that we see and inferring a specific kind of low entropy state. So it's not a completely circular argument, but it is a self-consistent argument.

1:01:44.4 SC: And it could be wrong, that's the important thing. We cannot guarantee with any metaphysical certainty that the early universe did have a low entropy, because we can't even guarantee there was an early universe, God could have created the whole universe last Thursday, we could be brains in a vat being taunted by an evil demon, we could be fluctuations from a high entropy state at early times, none of those ideas are things you can rule out on the basis of data once and for all. You can assign them low credence, you can do that, and then you can see, is that a fruitful thing to do? Can I make progress on the basis of doing that? And that's what we do with the low entropy of big bang. We say, well, if it was high entropy, then our existence is just a crazy accident and we know nothing reliable about the universe, and that's fine, but it's no way to go through life. You could say, or hypothetically, maybe the early universe had a low entropy. Then what follows from that?

1:02:40.2 SC: And what follows from that is a sensible picture of how the universe evolved and confidence that other implications that we draw about the universe are reliable. So the past hypothesis is called a hypothesis because you can't prove it, but making it makes everything fit together in a nice way, and it didn't have to. I mean, you could assume the past hypothesis and then go out and make an observation and see the universe looks like it was high entropy in the past, right. And that would be a reputation, but you can't prove it, but you haven't disproven it either, and it works in a self-consistent way. That's life. That's life as a scientist, you can never know the evil demon is not taunting you, but that's no way to go through the day. Siedhoff says, In listening to your Mindscape interviews, I've noticed that many of your guests have recently published a book. Furthermore, it seems clear from the interviews that you've almost always read or absorbed their book prior to the interview. Is reading these books included in the one day per week you've mentioned that you spend on the podcast? Yes, it is.

1:03:40.4 SC: But of course, some interviews require more reading than others, so it's not like it's... When I say it's one day per week, that's an expectation value, that's an average. Some require less. Some require more. And remember, one of the big motivations for me doing the podcast at all is that I like reading these books. These are things that I would like to be learning about anyway. This is one of the reasons why I think they're are fun conversations that we have, because it's not like I am a professional interviewer who has been told by some producer that this person would be interesting to talk to. The people I'm talking to are people who I want to be talking to. And they often have books because it's easier to get people in the podcast when they have a book to publicize, that's a very natural thing to do. And so it's a mutual back-scratching kind of arrangement. And finally, most importantly, it's possible that I have not read every word of the book. I do try to get the point of the book. Sometimes I will read every word, but other times the books are long, I'm in a hurry, I'm trying to get enough of a point that I can ask relevant questions.

1:04:48.9 SC: The line you want to walk on here is I don't want to sort of tell the guest what to say it, so it's not like I wanna go, okay, now we have to talk about this, and we're talking about chapter three now, and there's a whole outline before we start, but I also don't want to miss something really interesting. So the real reason for reading the book is that there could be really juicy stuff in there that I don't want to neglect to ask about. In principle, you don't need to read anything because you're having the person on. You can just ask them the questions, they're right there in front of you. You have Marshall McClure in there, you can ask him questions. But the real worry is that you're gonna miss something interesting that they have to say, so I always tell my guests, look, you know more than I do about this subject, so feel free to talk about the interesting stuff, but then I also try to give them the space to do that by asking questions that are inspired by the stuff that is in their books.

1:05:51.9 SC: F Sub-H is the name given to this person who says, just to make things clear in my head, when you say the universe can fluctuate into existence, I think you said that, does this really mean that fluctuations in De Sitter Space can cause elementary particles to randomly come together in a configuration that creates an entire universe just like ours? If so, isn't this then the answer to the question about the origin of our universe, how can we disprove that it wasn't created in this way?

1:06:18.9 SC: Well, so there's a bunch of ideas in there, just to clear things up. So the first thing to say is, yes, it is possible that fluctuations in De Sitter Space can cause elementary particles to randomly come together in a configuration that creates an entire universe just like ours. This was something that people have talked about. I think Dyson Kleban and Susskind, Lenny Susskind, former Mindscape guest, was one of the first papers to write about this, and then I wrote a paper with Anthony Aguirre another Mindscape guest and Matt Johnson, that went into more details about what that would actually look like, this process of fluctuating in... A whole new universe into existence.

1:07:03.3 SC: It's also possible that it doesn't happen. Even in De Sitter Space, the nature of the fluctuations is not completely clear thing, so do you get in De Sitter Space, the kind of fluctuations that give rise to a universe, or to brains, or to people or whatever, unclear. Or, what I should say is, it depends on other details. You can't just say, De Sitter Space, therefore fluctuation. So you have to say, De Sitter Space plus these extra hypotheses, and then you'll know whether or not there are those kinds of fluctuations or not. It's a model-dependent kind of question. The final thing to say is, no, this is not the explanation for how our universe came to be. That's exactly the Boltzmann brain problem. If you're in a situation where you do have De Sitter Space, De Sitter Space last forever, okay, it's eternal to the past and future, and if you have fluctuations that randomly create universes, you will also always have fluctuations that randomly create smaller parts of organized sub-systems, people, planets, galaxies, whatever, and you're gonna make individual people or planets way more often than you're gonna make entire universes, so that's a theory that could explain the origin of our universe, but that theory makes predictions, and the prediction is, we don't live in the whole universe like that, we live in a much, much, much smaller universe than you would have gotten that way.

1:08:27.9 SC: So the theory is not compatible with how we think the universe actually works. I don't wanna say it's ruled out by the data for complicated reasons dealing with cognitive uncertainty that I talk about in my paper, Why Boltzmann Brains Are Bad. But the short answer is, we do not think this is a plausible explanation for where our universe came from. It doesn't fit other things we know about the universe. Trevor Villwock says, priority question. Remember that every Patreon gets, once in their life, the ability to ask a priority question that I will promise to try very hard to answer. I don't necessarily promise that the answer will be satisfying, but at least I'll give it a shot. So the priority question is, shouldn't we consider the default rational view of an afterlife to be that life repeats eternally rather than simply ending at your death? And then Trevor gives a lot more explanation of what he means by that, Patreons, you can read that in the comments where the questions are left. So no, I don't think that is true at all, so let me just... I'm not sure that I got the scenario that Trevor was outlining in his question, so let me at least try to say something useful by saying what I think my view is.

1:09:45.2 SC: I think that life is a process, I don't think it is an essence. Okay, so the distinction being drawn is between a literal substance versus a way of describing the dynamics of some kind of other substance, right. So this is not a very spooky kind of distinction. When you think about energy, energy is not a substance, either energy is a property that substances have. Substances can have kinetic energy or potential energy or whatever, but it's a thing that has that energy, an atom, or a rock, or a planet, or what have you, and the energy is the way we talk about it, the way we characterize that physical thing. I think that life is like that, although unlike energy, life is not conserved. Life is a temporary process. It begins at the beginning of life, it ends at the end. The boundaries... And actually, this is gonna be a theme, this is gonna recur later on in the AMA. The boundaries of when life begins and when life ends are not necessarily cut and dried.

1:10:50.0 SC: We're thinking about this, of course, because the question of abortion and Roe versus Wade is on the docket right now in contemporary politics, so that's not what this particular question is about. So let's think about the other end. Let's think about death. Death is not cut and dried either, you can have different parts of your body shutting down at different moments in time, you can have... You can be dead by all scientific criteria, but if you've recently died, maybe you can be revived because life is not a substance that left the body, some sort of élan Vital that was located in your body and went somewhere else. It's a process, and you die because the process stops. And if your body has not broken down too much, doctors can come in there and quickly restart the process, and it keeps going, and you can keep your memories and the whole bit, okay? But eventually it will stop and the fact that the boundary, the moment when you actually die is not cut and dried is completely compatible with the fact that it does stop. So I started saying, this is gonna be the theme of the AMA, just because a certain category or idea is not precisely exactly rigorously defined does not mean it's not super-duper useful.

1:12:07.6 SC: So whether or not a thing is alive or dead might not be super-duper, rigorously well-defined, but it is still pretty clear in some circumstances, and therefore it is a very useful concept to have. So if a person dies, and then they're cremated and their ashes are spread around, they're dead, they're never gonna come back, there's no necessary sense in which they repeat eternally. That's the sort of physical biological aspect. The cosmological aspect here is does anything repeat eternally? And you know the answer there is, maybe, maybe not. There's certainly no reason to think that anything in the universe repeats eternally. We don't know whether the universe had a beginning, whether or not it will have an end, and if it does last forever, if the universe is eternal, it's, again, possible that it's an eternal and a repetitive sense, but it's also possible it's eternal in a non-repetitive sense. So my favorite... Sorry, to those who've heard this before, my favorite example is the integers, the numbers zero, one, two, three, four, also, minus one, minus two, minus three, minus four, an infinite number of integers, and they never repeat. Even though there's an infant number of them, there's only one of them that is the number five. And the universe could be like that, it could be eternal, but no one moment ever repeats.

1:13:28.6 SC: And in fact, I think there's very good reasons to think that if the universe is eternal, it would be like that. And again, it's a complicated story, but it goes back to similar Boltzmann brain kind of considerations, if the universe recurred, if there was a finite number of things that could happen and they happen in infinitely many times, probably, again, not certainly, but probably they would happen more often than not in low entropy, fluctuations, in small fluctuations away from thermal equilibrium, I should say. Our universe is a huge distinction, distinction, divergence from thermal equilibrium, so it's not the kind of universe you would expect to live in if we lived in a recurring universe. I talk about this a lot more in the book, From Eternity to Here, if you wanna know more about that. Carlos Nunez says, Between the view of Francis Fukuyama, that all the world will progress toward a global capitalist democratic order and Samuel Huntington's view of warring civilizations in perpetual conflict due to profound cultural differences, which position seems more reasonable to you? Can non-western civilization such as China ever embrace Western democratic values.

1:14:38.1 SC: So the answer is I don't really buy into either one of those parts of the dichotomy, so certainly not the idea of Fukuyama, that the world here on Earth is progressing toward a global capitalist democratic order. And I don't wanna completely poo-poo the idea. I get it, there is not a teleology in the progress of history, but there are trends that go on for a long time. And I think it's okay to imagine that the universe is becoming more globalist, capitalist, democratic than it was before. But I also think that there is some inherent instability and some churn. I think that it is probably a mistake to think that there is in human culture ever going to be a permanent or quasi-permanent equilibrium.

1:15:33.8 SC: Especially, but this is hard to say. Maybe there will be, but when you're in... Human beings are coupled to technology and the environment and things like that. What I should say is human social and political systems are coupled to the environment here on Earth and also to technology. All the ways in which we can change our environment through science and engineering and so forth, and both the environment and technology are in states of rapid change right now, they are nowhere near equilibrium, we're dumping greenhouse gases and we're killing off species and things like that, and we're developing new technologies, new ways of changing our everyday lives, and so it would be very, very, very, very surprising to me if human culture and politics somehow stayed constant in a world where the surrounding boundary conditions were changing rapidly, were changing dramatically. And even if the surrounding boundary conditions were more or less constant, it's still very plausible to me that human societies would be changing.

1:16:41.5 SC: I think that a characteristic of life at the very basic level of what it means to be a living being is that it is an out-of-equilibrium process, we take in low entropy energy from the sun, we give it back to the universe, and we sort of survive on that dynamical process. And that's not directly related to politics or culture, but it's an analogy. Human culture is also an out-of-equilibrium process. It is not something you should expect to settle into an equilibrium, it's not that it can't... There are quasi-equilibrium processes that are, in some sense, dynamical. A hurricane can last for a while. The Great Red Spot on Jupiter is a better example because hurricanes don't last for very long, but the Great Red Spot is not some big rock that is red, it's a storm, it's a dynamical thing, the individual molecules move in and out of the Great Red Spot, but the pattern lasts for a long time, even though the thing as a whole is out of equilibrium. So you could imagine human culture being like that, taking in information and energy, giving it back out, and yet maintaining some kind of overall homeostatic configuration, but I don't see it. I don't see it happening right now.

1:17:57.3 SC: So certainly, I think that it is a very, very short-term blinkered perspective to think that the very, very recent changes in human culture are gonna last forever or even for centuries. I think that that's unlikely. On the other hand, Huntington's view of warring civilizations seems to me to be in some sense a flip side of the same short-sighted coin, to take current conditions and extrapolate them to forever. Even if there's churn and change and dynamism in human society, the idea that it will just take the form of the kinds of conflict between cultures that we have right now just seems wrong to me. Cultures change, they intermix, and we're clearly in a stage of human development right now where those changing and mixing are rapid and important. So to somehow take some aspects of that and imagine that they're frozen in and will last forever seems... Even if I don't know what the actual future is gonna be like, I'm gonna bet that it's not gonna be that. That seems very improbable to me.

1:19:05.1 SC: Now, you tacked on an extra question. The rules are only one question, but again, you know I always bend the rules, it's my podcast. What can you do? You can't stop me. Can non-Western Civilizations, such as China, ever embrace Western democratic values, and I just can't let that one go because I think that there's many assumptions being smuggled in here that I don't want to grant. I think it's just completely bogus to think of democratic values as Western and to contrast them with Eastern or Chinese or Asian values to any sense, in both senses, in both the sense that democracy is Western, or that Western is democracy. Anyone who knows the history of Western civilization, if you wanna call it that, mostly non-democratic over the course of history, right? Some including very recently, including right now, some very brutal autocratic cultures, societies, governments, there's democracies there, but that has not been the consistent story for the last 3000 years in the Western world by any means. It's been mostly dictatorships and autocracies in the Western world, and if you have not heard it or if you don't remember, I encourage everyone to go back and listen to the podcast I did with David's Stasavage about the origin of democracy. It is not an exclusively Western idea.

1:20:30.2 SC: There have been democracies all over the place. Now, what's going on, of course, is that the Western world, the European world and its sort of cultural descendants did catch on to the Industrial Revolution and the Scientific Revolution and the Enlightenment first, and that managed to let them colonize the world, and there's a very fascinating inter-relationship between those changes and the changes of democracy, and I don't even know what to call it. The age of reason and enlightenment and the political side of things. I'm sure that there's a well-known phrase that I'm just forgetting right now, I've not gotten a lot of sleep because moving across the country is tiresome, but the point is, there's been, in the last couple of centuries, a resurgence of democratic governance in the Western world, yes. To think of this as somehow culturally alien to Asia or something like that is just entirely wrong in my view, and I think that it's clear why people try to do it.

1:21:35.2 SC: The people who try to do it are the autocratic dictators that are ruling countries in China and elsewhere, they want to provide an excuse for not being democratic 'cause they like the power that they have, and there are certain apologists that they have who are not them themselves, but I think it's entirely bogus. Maybe... I'm certainly willing to imagine there is a relationship between culture and governance. I'm certainly willing to imagine that different cultures will settle on different kinds of ideal forms of government, but Japan seems to be doing pretty well with democratic governance, even though it's not a Western country. I think that the idea of democracy can be implemented in many different ways, but its fundamental foundation is the idea that the authority comes from the will of the people, not from some special elite that gets the right to govern. And I think that that is a universal thing. I think that's a human thing, I don't think that's a western thing at all, so I have no trouble whatsoever in thinking the countries elsewhere in the world can embrace democratic values, even if not western values.

1:22:48.6 SC: Okay, I'm gonna group two questions together. I really... By the way, anyone listening, I love doing these AMAs. They're something to do, it takes time, but it's well worth it, and part of the delight for me is just randomly popping from topic to topic and talking about it. It might not always make for the most coherent listening experience, but it's fun for me. So thank you for indulging me in this. So here are two questions grouping together, one from Justin Walcott that says, the fine-tuning argument says that the odds that the conditions of the universe that supports life randomly occurring is 10 to the minus 223. But the odds that God would create a universe that supports life is 100%. Therefore, God is exponentially more probable than no God. The fine-tuning argument itself feels flawed by picking an arbitrary phenomenon of life and applying a success criterion for a PEP theory after the fact. I can't put my finger on it, but this feels like a dishonest research technique, much like p-hacking, is it?

1:23:46.0 SC: And then Mark Schoyan says, "I'm currently reading David Chalmers Reality+ and enjoying it. I came across a statement though that strikes me is incorrect, and I'm curious about your thoughts on it. He says that a weakness to the multiverse as a solution to the fine-tuning problem is that it only bumps the problem upper level because the multiverse would have to be fine-tuned to allow for universes suitable for life to come into being. I am under the impression that all we need are String Theory and inflation, and we end up with universes with a variety of physical laws, and we don't need some set of laws for the multiverse that determine how many and what sort of universities we get. Am I thinking about this incorrectly?" So I'm grouping these two questions together 'cause they're both about fine-tuning and the multiverse and things like that, but these are good, these are better than average questions about fine-tuning in the multiverse. So I like them. Justin brings up this idea that the probability of getting a universe that supports life is 10 to the minus 223, and I've seen numbers like this bandied about before, and they're completely entirely bogus.

1:24:50.8 SC: Sorry about that, people who like these numbers. The way they get them is they take some parameter in the universe like, I don't know, the fine structure constant or whatever, the ratio of the neutron mass to the proton mass, a million different numbers, the cosmological constant, not a million, but a larger number of numbers, and then they completely pull out of their butts some probability distribution for these numbers. And by the way, bless them for doing that because particle physicists do it all the time too. Particle physicists who have no interest in fine-tuning in the sense of design or God or whatever, they make up probability distributions for no good reason, and I think that this is a disastrous flaw in the entire discussion of these issues, and this is one of the reasons why we need better philosophy here, because these physicists and these theologians are just making shit up, and we should be better than that.

1:25:46.8 SC: So anyway, they invent a bunch of probability distributions for numbers to have, and then they guess or try to estimate the range that those numbers need to have in order for life to exist, and the ranges are small, and they multiply together a whole bunch of small numbers, and then they get this ridiculously tiny number, 10 to the minus 223. So there's a lot of elementary mathematical mistakes that they make in doing that. Even besides inventing these probability distributions, they often then treat these probability distributions as independent of each other. The probability distribution for defining structure constant, and then the probability distribution that the temperature of the air on the earth will have a certain value or whatever. But these are not independent of each other, so you just can't do that. So the sort of a very simple mistake that is being made. But the bigger mistake is just this idea that we know the probability of the universe like ours. There's two big mistakes, one is that we know that probability of the universe like ours.

1:26:52.1 SC: Number one... Sorry, there's sort of sub-mistakes. The one big mistake is we know we can calculate the probability of a universe like ours. The sub-mistakes are, number one, we can calculate the probability distribution for individual constants, but number two, that somehow a universe that is more or less like ours, but with these constants is the only kind of universe that we could have. A universe that is a multiverse kind of universe is completely compatible with all of the observations we have. Maybe in multiverse, it's a 50% chance. And if that's true or maybe it's a 1% chance, whatever. But anyway, there's zero reason to think the multiverse has a chance of 10 to the minus 223, so it's just an entirely bogus kind of calculation done by people who already think that they know what the right answer is supposed to be. And then the other thing that they're assuming is that we know the probability that life can exist. And here, I do think that there are some conditions that you could imagine the universe having that would be so incredibly hostile to complex structures existing at all, that all of us would be willing to grant that, yeah, life could not exist there. But I don't have any confidence at all that in a somewhat complicated universe, you can pinpoint whether or not different kinds of life could exist.

1:28:19.0 SC: These calculations generally want to make life really, really, really human-like and leave no room for other very, very different kinds of life like complex structures. So I think that these calculations are completely entirely wrong-headed. Now, I also think the fine-tuning argument is wrong for other reasons, for better reasons. This is not the real reason why the fine-tuning argument is wrong. The real reasons why it's wrong are, number one, God wouldn't need to fine-tune the universe for life to exist. The fine-tuning argument only works if you're already a naturalist or a physicalist about life, because if God existed, then the physical universe could do anything at all, and life would not be purely physical. If life is just an emergent phenomenon from complex structures in the universe, that sounds like you're already a naturalist there, and so what you're calculating is the probably that a naturalist universe would exist that allows for the existence of life. But God can create life in any universe at all, so that's a really dopey argument for the existence of God.

1:29:27.1 SC: And the second huge problem with it is that as a good Bayesian, you are not allowed to just say, Oh, I have one piece of data. Life exists, and I'm gonna calculate the relative probability that this happens with God or without God. If that were the only thing you cared about, the only data you had, then I would actually be willing to buy that the preponderance of that data is more favorable towards theism than naturalism, because if that's... I get that you could invent a version of theism where God would definitely want life to exist, but then you gotta be consistent, then you gotta be able to say, okay, what other predictions should you make for what the universe should be like under this theistic theory? And there, you start making predictions and you find a universe very, very different than what we live in. So that idea is more or less ruled out by the data if you're a consistent good bayesian. So anyway, fine-tuning argument, entirely bogus. I have said, and I will stand by the idea that it is the best argument for the existence for God, because it plays by the rules, it's an argument that tries to be Bayesian, tries to say, well, we have some priors, we update them and we draw conclusions on the basis of that. Unlike these ideas that just let us sit in our arm chairs and just reason into the existence of a transcendental being, like a necessary being or something like that, which those are even more bogus.

1:30:57.9 SC: So it tries to play by the rules in that sense, but I think that it's still not very good at it. And then to Mark's question about David Chalmers' objection to the multiverse, yeah, I also disagree with that completely, and I agree with David about a lot of things, but I disagree with him about a lot of things also. I think that he is... I don't remember reading that section of the book, to be perfectly honest, Reality+, which is a great book, by the way, you should definitely check it out if you haven't. It's extremely readable, even though it's discussing a high level of philosophical things. It's a fun read, to be honest. He really did a good job at writing a very accessible book. But this argument that the multiverse has to be fine-tuned, I just don't think is right. The whole idea of a multiverse is there is some simple generating principle that creates all sorts of different universes. Maybe that generating principle is the combination of string theory inflation, that's the popular one that we're thinking of now, but it's... Again, it's like the integers. I don't need to invent an infinite number of numbers, I need to invent a single number, zero, and a single operation that adds one, okay? And then boom, I have an infinite number of numbers. And the multiverse is like that, it's a very simple process that gives rise to all these universes and so no necessary fine-tuning is involved.

1:32:24.2 SC: Bruno Tatchera says, there have been some news about a new particle discovered, the axial Higgs boson or the magnetic Higgs, I read the abstract of the article but it was out of my league. Can we get your take on it? Well, yeah, I'll give you a mini-take here because I think there is some clarification. Even though I'm not a super expert on what is actually done, I am an expert on what was not done, which was discovering a new particle. The old Higgs boson that we discovered before is still the old Higgs boson. That's the only particle that we have that fits that description within the standard model. But here's the thing, the Higgs is part of a mechanism, the Higgs mechanism, and the mechanism is you can have a symmetry in some kind of theory like gauge symmetry, that is to say a way of transforming fields where everything remains unchanged when you do the transformation. And the Higgs mechanism is some other field that transforms under that symmetry, it gets a non-zero value in empty space, the so-called vacuum expectation value. And when that happens, the symmetry looks like it's been broken, so we call that spontaneous symmetry breaking or non-linearly realized symmetry or something like that, but anyway, you wouldn't notice the symmetry if all you knew was life inside that vacuum state, and that was the case for particle physics.

1:33:41.8 SC: You required some imagination, some IQ points to realize that there was a symmetry there, even though it was apparently broken. But this idea of a Higgs mechanism is a pretty general idea. There's the actual implementation of that idea in the weak interactions of Particle Physics, in the this so called electroweak theory, that breaks down the SU2 cross U1 symmetry of the standard model down to just U1, which we see is electromagnetism, but then you can have similar mechanisms in all sorts of other systems.

1:34:12.6 SC: And this so-called new particle that was discovered, it got a little bit of press and coverage recently was of that form, it was a completely different system, it was a condensed matter system, nothing like the fundamental particle physics of the electroweak model or anything like that, and they saw a different kind of Higgs mechanism that they call the axial Higgs boson or the magnetics Higgs, whatever. It's not a new particle in the standard model. And honestly, the set of people who were responsible for this news story are... They were bad. They misbehaved. I don't know whether the blame goes with a press office at a university or the scientists themselves or whoever, but you can say things that are technically true in a press release, but you know, if you're honest with yourself, are going to be misinterpreted by everybody, and this is exactly in that league. When you say I've discovered a new Higgs boson, people are gonna naturally think that you've discovered something extra beyond the standard model of particle physics or a new part of the standard model, and that's certainly not what happened here.

1:35:16.6 SC: So that was a little bit of bad PR mongering, I would say. Nicholas Chapman says Boltzmann's formula for entropy is entropy is proportional to the log of the number of microstates for a given macro-state. Using this definition, the Second Law of Thermodynamics, then just becomes almost a tautology that more likely states are overwhelmingly more likely to occur in the future. How then is the second law useful in any way when defined like this?

1:35:41.5 SC: Well, remember, so you've already cheated, Nicholas, by saying second, that states with higher entropy are more likely to occur in the future. Who said anything about the future? Boltzmann's formula doesn't say anything about the future, it just says that the entropy is proportional to the log of the number of micro-states, okay? Anything you say about the future, if that's the only thing you know, Boltzmann's formula, you should also say about the past. But in that case, you don't, in the real world, think that entropy was overwhelmingly likely to be larger in the past, so you have to augment Boltzmann's formula by the past hypothesis as well as some more specific technical requirement that you are not in the highly tiny, but nevertheless accessible kinds of states where entropy would decrease even though it is not forced to. You can sort of have a conspiracy and all the molecules in your gas or whatever to be pointed in an entropy decreasing direction. So there is content to the Second Law of Thermodynamics. It says not only is it likely that entropy will increase from today to tomorrow, but then it was smaller yesterday, and that's not gonna pop out of Boltzmann's formula all by itself.

1:36:56.0 SC: Richard Kenan asks a priority question. Do you think it is acceptable to come to conclusions about other political and sociological situations like those in China, for example, without conducting the proper research and having evidence to back up most people's claim that a liberal democracy is the only civilized way to govern a population? It seems to me that almost everyone gets their information on places like China from mainstream media who are known to lie and to be biased in the extreme when it comes to China in particular. It is also known that the US government has a budget of millions to fund anti-Chinese propaganda, when surely that money could be better spent on US citizens welfare. So there is a lot going on in this question. Do you have a right to have an opinion without conducting proper research? Everyone has a right to an opinion or to come to conclusions but the better way of saying it is, how plausible or believable or credible are the conclusions that you're reaching? And clearly the more evidence you have and the more proper research you've done, the more credible your conclusions are, there. I could stop talking there, that would be an answer to the question. But clearly this is what one of those things that in the lingo of the trade is more of a comment than a question.

1:38:08.4 SC: And trying to make a point to rather ask a question. And the point being made, I don't really agree with at all. So as soon as, just for future reference, not anything personal against you, Richard, but when people start complaining about the mainstream media, I almost immediately know to not take seriously what they're saying, because of course, the mainstream media is not always right. Of course, the mainstream media makes mistakes, of course, it has its biases, and it has systematic biases where you can critique it for very good reasons, and I've done that myself. But guess what? All the non-stream mainstream media also has biases and problems, and typically more biases and problems and predilections for making mistakes than the mainstream media does. So the point is not mainstream versus non-mainstream media, the point is for any given information source, how credible is it, how likely is its way of telling a the story being to be influenced by its biases and its prejudices and so forth. And I'm sure that the United States government has propaganda, I'm sure the Chinese government has propaganda, I'm sure that Taiwan has propaganda and Russia has propaganda and all of these things.

1:39:18.9 SC: I don't really think that's relevant to this question, and especially when surely that money could be better spent on US citizen's welfare. This is just not a very rigorous comparison of how we should allocate our budgetary resources. I think we can do a little bit better than that. Finally, the claim that a liberal democracy is the only civilized way to govern a population, I very rarely, if ever, hear people say that in those exact words. This seems like a little bit of a twisting of what people say. I do think it's the best way to govern a population, and I'd be happy to defend that. Certainly, it's better than what China does. China is oppressive autocracy. I think that's just true. I've been to China. There is never an election where the Communist party wants to win and doesn't. That just doesn't happen. You can't get certain websites from the outside world, you cannot learn about what happened at Tiananmen Square, if you're in China.

1:40:15.3 SC: That is not the kind of society that I want to live in. For all of the problems that we clearly have here in the United States, we have more freedom of information, more ability to choose our leaders. Just because things are not perfect here, it doesn't mean they're not better than other places. And there are places that are worse than China also, so you have to actually take some care to get things right. On that, I completely agree. Yugal Carmel says, is it possible or reasonable to formulate a version of Many-Worlds quantum mechanics without actually believing in the Many-Worlds, in other words using entanglement decoherence to explain measurement collapse and dissociated probabilities as Many-Worlds does, but being agnostic about whether the other branches exist? Well, I don't think that being agnostic is part of a scientific theory. That's just not really a thing. Scientific theory is a set of equations and sort of interpretation for what those equations are telling you. If you believe in the Schrodinger equation, unitary evolution as Many-Worlds posits, and you believe that the wave function is realist, that it somehow represents aspects of reality, and you believe there's no violation of the Schrodinger equation, then the theory predicts the existence of the worlds.

1:41:31.3 SC: So either you believe that theory and you believe in the worlds or you don't believe in the theory, so you have to change the theory somehow. You can't just be agnostic about what the theory is predicting. You either think the theory is right or you think it's wrong, or of course, you have some credence for whichever thing, but there's no stance where you say, I think the wave function is realist, and there are no hidden variables, and there's only Schrodinger evolution. But I choose not to believe in the other worlds, that's just an inconsistent attitude to have.

1:42:01.6 SC: Joey Colbeck says, Does the cosmic microwave background have a spectral signature? Yeah, sure, absolutely. If you're my age, there was a big deal when I was in graduate school about the spectrum of the cosmic microwave background. The spectrum, just to go way back to the basics here, by spectrum, we mean taking a set of electromagnetic radiation or whatever, and decomposing it into the different amounts that are coming to you from different frequencies or wavelengths. So you can absolutely do that with the cosmic microwave background and guess what? You get an almost perfect black body spectrum. That is to say you get the same kind of spectrum from a system and thermal equilibrium at a certain temperature, just giving off radiation. And that temperature is 2.7 Kelvin.

1:42:48.3 SC: So that is the spectrum. Sorry, I should back up again a little bit because people use the word spectrum with respect to the microwave background in two very, very different senses. A spectrum is just whenever you have a set of waves, you talk about the different amount of waves with different wavelengths, okay. So in the actual microwaves, in the cosmic microwave background, those are waves, they have wavelengths of order of centimeter and you can decompose them into different wavelengths, and that's the spectrum of the cosmic may background, it's a black body. But then you also have density perturbations in the early universe, right? Those are the perturbations that give rise to the Anisotropy of temperature in the cosmic microwave background and eventually grow into galaxies and structures. And so as physicists tend to do, we instantly think of those Anisotropy and perturbations as a set of waves, as a set of modes in Fourier space, so you can talk about the amount of perturbation at a certain wavelength, and another wavelength, and another wavelength, and altogether, the amounts of perturbations with different wavelengths are the spectrum of perturbations of the cosmic microwave background.

1:44:03.9 SC: So that's not a spectrum of wavelengths of light, it's a spectrum of wavelengths of perturbations, and rather than centimeters, we're talking about millions of light years or whatever. So let's not talk about the perturbations, that's not what we're talking about. The spectral signature of the CMB itself is almost a perfect black body. So to go back to where I started, when I was a graduate student, there was a hint from a telescope, from satellite experiment, actually, that the microwave background was actually not a perfect black body. There's a little deviation from it. And on the one hand, this is completely expected, you wouldn't expect any real thing in nature to be an absolutely perfect black body, because there's various ways to deviate, something get in the way, something... There could be foregrounds that mess it up, there can be processes that push it out of equilibrium, exploding stars or whatever, energetic things happening in the early universe. And there was a claim from... Was it even a satellite or was it just a rocket? I think it was a rocket-launched observatory that didn't actually orbit. But anyway, the claim was there was a slight excess at long wavelengths in the microwave background spectrum, and an enormous amount of effort went into trying to explain this excess.

1:45:16.9 SC: And I even have friends who did PhD thesis trying to explain this excess, and it went away, it was not there, it was an artifact of a bad experiment, it was a mistake. They were basically observing their own rocket exhaust even though they tried very hard not to do that. And then when the COBE satellite went up, the very first thing it did was to observe the spectrum of the CMB, and it was a perfect black body as far as it could be observed, so I think that even though it needn't be an absolutely perfect black body I believe it's still true that the cosmic microwave background spectrum is the best black body that we know of in nature. Someday we'll find a deviation from it, but not yet.

1:45:53.3 SC: Dom says, do you think physics can ever be finished? Well, it depends on what you mean by finished. I do think that we can find the fundamental laws of nature. There's a way the universe works, and we can find it. I don't see any obstacles to that. Maybe we won't, I didn't say we will, but I think that they're there, and I think that we have the intellectual ability to do it in principle. Now, there's more to physics at the fundamental laws. The fundamental laws might be a small compact set of rules, but the way that those fundamental laws manifest in the messy macroscopic world, in the emergent world, if you like, is of almost infinite variety, maybe not technically infinite to a mathematician, but really, really complex. Like in some sense, biology is a subset of physics. So to understand all of physics means understanding all of biology, and I don't think that's gonna happen at least any time soon. So I don't think that science will ever be finished, although I do think that it's possible that the fundamental laws will be figured out. And then what physics will be doing is mostly figuring out the implications of those fundamental laws, and that's what most physics is even today. So the chances of finishing that at any reasonable time frame are extremely tiny indeed.

1:47:09.0 SC: Seamus Maclennan or Seamus Maclennan says, I would love to hear your thoughts on Karl Popper's tolerance paradox. Personally, I'm constantly frustrated by friends who are on the liberal left, who constantly advocate for free speech, acknowledge that there is a line somewhere but refuse to define where that line is. So there's a couple of things going on here as usual. The paradox of tolerance from Karl Popper, we talked about in the Mindscape episode with Teresa Bejan. I encourage you to listen to that. And neither one of us are fans of Popper's tolerance paradox. The tolerance paradox is you try to be a tolerant society, but that means you let anyone have speech rights, free speech, but then some people who wanna speak are not themselves tolerant. So by being tolerant towards intolerant people, you are opening up the danger of them overthrowing your society and making it no longer tolerant.

1:48:06.6 SC: And the answer is, yeah, you are opening up that danger, but maybe that's a danger, a risk that is worth taking. I believe that in most cases, not all cases, but in most cases, when it comes to the kind of discourse that is purely about political process or political theory, even if it's intolerant, even if it's racist and sexist, and tyrannical and authoritarian, even if it's like pro-fascism, I think that the way to deal with that, if you want to fight against racism and sexism and fascism, etcetera, is not to prevent that kind of speech, it is to address the issues raised by that kind of speech. I think there is a more robust way of being anti-racist that you get to by knowing what it would mean to be racist, by understanding why. As I said before, with discussing with people who you disagree with, understanding why they would ever think that is actually important, not because you're gonna be persuaded by it, but because you wanna persuade them and you wanna understand your own point of view better, and you do that by listening.

1:49:12.6 SC: I once wrote on Twitter... Sometimes I write things on Twitter that I think are perfectly benign and why am I even bothering, and then I get a lot of abuse for it, and I said... I forget exactly the phraseology, but it was along the lines of, If you want to understand a point of view, listen to it from the perspective of the people who believe that point of view. Don't get all of your information from the skeptics of that point of view, 'cause in that case, you're probably not getting the full story. And people really, really objected to that, like what if the point of view is bad. Especially if the point of view is bad, that's when you should get it from the horse's mouth because then it's a puzzle, why do people have these bad points of view?

1:49:53.4 SC: Well, you could try to just think of them as terrible evil people, or you could listen to what they're saying. And so I think that Popper was wrong in that way of framing the issue of what to do with intolerant points of view. Now, having said that, I'm not an absolutist about free speech, I think I'm probably... I have a wider scope for it than a lot of people do, but I do completely agree that there should be limits on it. There's speech that directly leads to violence and bad behavior, and I think it's perfectly sensible to restrain that kind of speech. It's a much more down-to-earth practical set of considerations that come into line, and that makes sense. Now, I think that the worry here about defining exactly where the line is, this goes back to what I said about the theme of this AMA, where I'm going to be arguing in favor of slight amounts of ambiguity and in precision. Yes, it is hard to draw the line between speech that is truly dangerous in the sense of people are gonna get hurt if you let this person do this versus speech that is just uncomfortable and you don't like it.

1:51:04.5 SC: You would like to be able to draw that line very clearly, but you can't. That doesn't mean the line isn't worth drawing. And in some sense there's two things to say about this, and they're both slightly tangential, but maybe illuminating. Remember the podcast we did with Teed Wen, where he talks about the seductive nature of clarity, the reason why so many people like games, which is fine, but also why they like conspiracy theories, which is bad, is because of the false sense of clarity. A game has rules and you know what they are, you know what's right and what's wrong, what's doable and what's not doable, a conspiracy theory explains everything, you're no longer stuck saying, Oh, I don't know why that's the case. You're like, No, here it is, The conspiracy made it true. That is a very seductive property for explanations to have, but that doesn't make it right. And sometimes the best explanation we have is not perfectly clear, there's some fuzzy boundaries. And that's not a flaw, this is just a necessary aspect of certain kinds of explanation. The standard thing is, if you have some high-level emergent phenomenon, something like a chair.

1:52:11.9 SC: Okay, everyone agrees that the thing I'm sitting on right now as a chair. Were you to see the object I'm sitting on right now, you would all agree, it is a chair. Now, if I remove an atom from it, Is it still a chair? Yeah, you would not even notice that I removed... I probably have removed an atom from it just by like rubbing my fingers on it, okay? You don't know. But if I keep moving atoms one by one, eventually I will get to a point where it is no longer a chair, certainly when I get down to one atom, it is no longer a chair.

1:52:41.7 SC: Okay, well, if you believe all of that perfectly reasonable stuff, which you should believe, tell me precisely how many atoms I have to remove to make it no longer a chair, you can't, because there's no such thing as the precise number of atoms. It becomes less and less chair-like, but there's no hard bright line between when it's a chair and when it's not. And the point is, that's okay. Just because you can't draw a hard bright line doesn't mean the concept of chair is not useful. The boundary between chair and not chair might be approximate and fuzzy and maybe some people draw it in different places, but it's still worth drawing because the concept of a chair gives you some handle on what happens in the real world. When I tell you I'm sitting on a chair rather than a sofa or on the floor, you know something you didn't know before. I gave you useful information. And that's a trivial example in the world of physical objects, but the same thing is gonna be true in world of politics and culture and things like that, there'll be a lot of principles that are sensible to follow, even though you can't draw the lines for them exactly. This is something you have to live with and do the hard work, don't demand clarity where clarity is not available to be had.

1:53:51.1 SC: Noble Gasse says, What are some ways to wrap our minds around the infinite regress problem as it pertains to the origin of the universe? It seems like an intractable problem because it's always possible to ask what came before that? If the current answer is that we just don't know, is it possible to say what you think an answer might look like? Yeah. So there is no such thing as the infinite regress problem. That is just not a problem. So I presume what you mean by this, is that if our universe is eternal to the past and future, then if you give in to the everyday temptation of explaining current events in terms of events that happen before, which is a very sensible thing to do in the everyday macroscopic world, causes precede effects, as we all know, but you can't extend that principle of causes preceding effects to fundamental physics or the cosmology. They just don't apply.

1:54:43.6 SC: The laws of physics aren't like that, the laws of physics are not causes and then effects, the laws of physics are, Here's a pattern. Laplace pointed this out, if you have data that tells you what's going on in the universe at one moment in time, you can push it forward or backward and explain either the past or the future, if you're Laplace's demon. None of us is Laplace's demon, but the laws of physics are Laplace's demon, they know how to do this. So there's no infinite regress problem because there's no principle that says I must explain the conditions at one moment in terms of previous moments. If there's infinite universe, there's an eternal universe that goes from past infinity to plus infinity, there's no initial time. There's no place from which the universe came, it was just always there. And that's it, that's the fact. It's always a mistake to quickly extend the kinds of reasoning you do in your kitchen, in your everyday life to fundamental questions about cosmology and the universe.

1:55:47.6 SC: Crabtree Luca says, Can you explain what the main disagreements are that cause some people to believe quantum entanglement is local versus non-local, and is it correct to say that within Many-Worlds, quantum mechanics is entirely local? So I think my point of view on this is not exactly standard, I think that quantum entanglement is simply non-local, full stop, 100%. Now, there are people who believe in Many-Worlds and will give you as a reason why that it seems local. And the way that they would say something like that is that there are branches and the things that are observed on any one branch or purely local phenomena. But I think that's... Number one, it's a little bit of a fast dance to define very carefully, what you mean by local, etcetera. There's certainly non-local correlations in EPR pairs and things like that. But more fundamentally, I don't think that locality is a big deal. I think that the whole discourse about locality, the idea that the world is made of things with locations and space and time is just completely overturned by quantum mechanics.

1:57:01.6 SC: Wave functions are not local. You cannot decompose a wave function in to the wave function here and the wave function there. The wave function is a function of every possible configuration of everything in the universe all at once. Space emerges from the wave function, not the wave function lives in space. So there's no reason to expect locality at all. So my attitude is a huge amount of effort and brain work in quantum mechanics and foundations of quantum mechanics is put into how can we live with the non-locality of quantum mechanics? Whereas, my attitude is exactly the flip side of that. Why is the universe local at all? Quantum mechanics isn't local, quantum mechanics is wildly non-local, but yet locality works pretty well in our universe. And of course, this becomes even more vivid when you have quantum gravity. In quantum gravity, if you imagine integrating over all possible space-time geometries, it seems even less likely that the universe would look local at all. But it does to a very good approximation. So I think that the question, the puzzle is the other way around, why do things look local at all, not how do we learn to live with non-locality.

1:58:13.5 SC: Alexandre Bates asks, do you think the US Constitution was a good basis for a democracy at the time of its writing? Do you think it remains a good basis for today's industrial technologically advanced US? Yeah, I think it's a great question. This is something that we really need to be thinking about, because on the one hand, yes, I think the US Constitution was... Honestly, it was amazingly good for its time. You think about the people who were sitting down to write that constitution and the expertise they had, the United States got really, really lucky with people who were both very smart and very well-intentioned. They didn't all agree with each other. Just go to watch Hamilton, the musical if you wanna see how people didn't always agree with each other. But they worked out compromises, and it actually was in many ways a very sensible system. But of course, it's not perfect, and of course, the flaws that it might have had from the beginning might get worse and worse, as the society changes, conditions change, the conditions in the country change.

1:59:19.1 SC: So on the one hand, I think that it would be absolutely possible to sit down from scratch and write a new constitution for the United States that would be much better than the one we have right now. On the other hand, I have very low credence that that would actually happen if we tried. Some person or some other very smart set of people might be able to do it, but given the current political climate, if we actually tried to get popular support for a new constitution, I bet it would turn out to be a disaster. So I still think that even though the Constitution has its problems, we should fix them through the existing mechanism of amending the Constitution. Even that's gonna be really, really hard to do because people disagree on how to do it. But I'm not in favor of a wholesale overturn of what the Constitution is. Now, there is a question... Sorry, I forget whether I wanted to talk about this here or...

2:00:17.8 SC: Yeah, maybe this... Let me ask the next question that we'll feed into Alexander's question also. So Vladimir Bellick says, you and hopefully most of us listeners are strongly in favor of democracy and want it to be preserved. However, as a thought experiment, could you come up with some genuinely reasonable arguments against democracy, arguments that you personally would find reasonable? So yeah, I don't think it's very difficult to find arguments against democracy, I don't think that there's a situation, it's even close to being the case that all the arguments are in democracy's favor. Democracy has its problems. One very obvious problem is instability. Democracy only works if people buy in to the idea that there should be a democracy. And this is one of the reasons that why we're in trouble right now, because increasingly we don't, here in the United States, people just don't buy in to the idea of democracy.

2:01:12.0 SC: Now, they might say they buy into the idea of democracy, or they might not. Some will just be explicitly against it. But I think a lot of people hear the word democracy and say yes, that means that I can vote for something, and if I get most people to vote on my side, then I win, and it gets implemented. But there's another part of democracy that is equally important, maybe more important, which is that if you lose, you have to live with it. That's the part that matters. It's not, if I win, I get what I want. It's if I lose, I still have to put up with it. Democracy is about living with people you disagree with. And the reason why it is, and this is sort of another issue with democracy other than instability, is that we have to take a whole bunch of people who agree on some things and disagree on other things, and imagine that among the set of things they agree on are the basic principles of democracy, including going along when you lose. And to find exactly that set of people that agree on enough to get along, but still can disagree on other things is a tricky line to draw. And so the instability problem is that if you get too many people who don't like democracy then in a democracy, they can end it. And there's almost no solution for that problem.

2:02:30.4 SC: If that actually happens, it's essentially too late. The only solution to prevent it from happening, to make sure that most people are on the side of democracy and to really educate them and convince them that they should be, and that's a non-trivial task, which I think maybe we take for granted a little bit. There's a more basic problem with democracy and that I think is the first issue that a lot of people come up with, which is that it's putting equal voice in principle. Of course, we know that in a world with inequality this is not actually true, but in principle, every person, every voting citizen has an equal voice, has an equal number of votes. But not every citizen is equally smart or equally in possession of expertise about difficult issues, etcetera.

2:03:16.9 SC: So there's sort of a technocratic, meritocratic point of view which says, Well, we should give expert people, smart people, talented people more of a voice than the average person. And I don't think that's right, but I get it. If you think of democracy as sort of a way to solve some science problem, or technology problem, or even a social engineering problem, then maybe, yes, maybe that's what you would want to do. But the real reason why people have equal voices in democracy are not because they're equally smart, but because we want all of their values and interests to be represented. So even if someone is not an expert on climate change or whatever, we want to be able to give them the right to say, I want this to happen, I wanna protect my values, my interests. Game theorists will tell you that this is just asking for trouble in some way, because there's a tragedy of the commons. People are not going to sacrifice their own personal well-being just for the sake of the greater good all the time. And that's an issue and we need to worry about it, but I never less think that it's better than the alternatives. And let me... And this goes back to Alexander's question about the industrial technologically advanced US. I think that's a more subtle question about democracy, which is whether it is compatible with modern technological, connected society.

2:04:42.9 SC: I think it is, I don't want be to alarmist about it. I think it can be, but when we did write down these constitutions 200 years ago or 100 years ago in other countries, the world was a different place. We talked to each other in different ways, the information flow, the news sources were different. And countries were smaller. Even if you thought... If you were a farmer in rural West Virginia, you might not have felt even years ago, very connected to what was going on in Washington DC, but proportionally, you had a bigger voice, 'cause they were just fewer people in the country. So when you have a country with hundreds of millions of people or billions of people in some countries, every individual vote doesn't count that much. And the reason why I'm relating this to technology is, as Daniels pointed out in my interview with them about everything everywhere, all at once in the multiverse, that what technology is giving us is a taste for the multiverse. Because we can look at other people's lives, we can see what's going on through whether it's instagram or TikTok or TV shows or whatever, the fact that other people are living very different lives than us is much more vivid and accessible than it used to be without giving us much more ability to live those lives ourselves.

2:06:07.0 SC: That's right. We can't all be Paris Hilton or the Kardashians or whatever. Most of us don't wanna be, but some of us do. And what all this gets to in some not very well-defined way is a worry that technology increases our alienation as citizens of a democracy, our feeling that we can't have a voice, we can't have an impact on what happens. The government is this huge remote thing. And there's a feeling of powerlessness that comes with this. And I think that this explains a lot of the rise of populism in the United States and elsewhere, a lot of people's willingness to vote for clearly anti-democratic figures, because they think that even if this person isn't very democratic, at least they're standing up for me. That's the feeling. I don't think it's true. But the feeling is something like that. And so it provides an opening for anti-democratic forces to say, "I will act in your interests in a way that this vast bureaucracy that is elected through many levels, etcetera, etcetera, is no longer responsive to your needs." And part of that does come from pure size. But part of it also comes from technology. And part of it also comes from our choice over the last half century here in the US to put up with way more inequality in some sense than we used to have. So it's not a fake feeling that people are being ignored. They really are.

2:07:36.3 SC: There's an uncomfortable number of people who are living lives that are less well off than their previous generations did. And that's not what we were brought up to believe. So I think there's a complicated combination of factors that are putting democracy in trouble. And some of them are curable. Some of them, I don't know. But anyway, I ended up worrying about the fate of democracy rather than give you arguments against it. But ultimately, there are arguments against it, Vladimir. Expertise could be bigger in a non-democratic society. It might be more compatible with modern technology. It might be more stable. And finally, you could act faster. This was a worry... Not a worry, but an argument people made at the beginning of the pandemic, which was that a functioning dictatorship could just implement measures to fight against the encroaching epidemic much more effectively than a democracy could, where there's all these babbling voices, and there's inertia, and things move slowly. And it's true, that a dictatorship can move much more quickly. But the thing is, it doesn't always move in the right direction. So again, I'm referencing the old podcast episodes. But with Henry Farrell, we talked about how democracy is actually a pretty good problem-solving mechanism.

2:08:51.3 SC: And I think we're seeing that in China and in other places now, where policies are being implemented which are too draconian to fight the pandemic, and that are making things worse off, overall. So democracy is not perfect at solving these problems. But I do think it's better than all the alternatives. That's where I would come down. Okay, where are we? Anita Tomasik says, "How much credence do you give to the concept of Boltzmannian immorality. Particularly do human... Do those human brain-like fluctuations called Boltzmann brains last long enough to give rise to any form of self-consciousness at all? So Anita, I'm not sure whether there was a typo and if you meant Boltzmannian immortality, but you wrote immorality, which I kind of like. So I left it in. And actually, I don't know exactly what either one of those concepts would be. But I think that it's a good opportunity to clarify some aspects of these Boltzmann brain fluctuations. So the idea is that if you live in a world where there are all sorts of random fluctuations lasting forever, you need both of those conditions to hold. If you live in a world without fluctuations or in a world that doesn't last forever, the whole argument goes out the window.

2:10:11.8 SC: And both of those are plausible possibilities. So we don't know if these fluctuations exist. But if they do, you tell me some condition on some local collection of matter. So that condition can be whatever you want. You can say, "I would like to have at least one kilogram of mass within a cubic kilometer." That's a condition. Or you can say, "I would like to have a planet with water oceans." That's another condition. Or you could say, "I would like to have a conscious creature." Or you could say, "I would like to have exactly me, a person that looks just like me with exactly my thoughts and beliefs about the world," whatever condition you want. In that fluctuating universe that lasts eternally, that condition will most often be achieved by waiting for a random fluctuation to bring that condition into existence, not by having what we call a thermodynamically sensible evolution from a low entropy initial state like our Big Bang to today. So when you ask a question like, "Do Boltzmann brains last long enough... " Well, some of them do. Some of them don't. Most of them don't. But some of them will. And you ask... There's two different kinds of questions you can ask. One is in the set of all fluctuations, what will most of them be like?

2:11:35.9 SC: And that's kind of how you're phrasing the question. But the worry, the problem of this scenario doesn't come from that. It comes from asking the other question, the other way, given some conditions, are they likely to be random fluctuations or is it likely to be thermodynamically sensible? And again, almost always, whatever conditions you wanna put, long lived, big, smart, handsome, moral, immoral, whatever, whatever conditions you wanna put in that randomly fluctuating universe, things like that are most likely to arise as random fluctuations. That is not to say that most random fluctuations will be like that. But most things like that will be random fluctuations. Therefore from whatever question you have, answer it in that basis, I would say. Flying Waffle says, "I don't have an actual question today, but I asked GPT-3," which for those of you who've been hiding from the internet is an AI program that has read everything in the world and constructs sentences and tells stories in the basis of prompts that you give it. So Flying Waffle says, "I asked GPT-3 to write a haiku in the style of Sean Carroll. And here are some of its best answers." And there's a whole bunch of answers. I'm not gonna read all of them. But just to give you a flavor, the very first one is, "He's a theoretical physicist and he writes about science and math. He's also pretty funny."

2:13:01.1 SC: So I love this as an example. Because it's a wonderful example of how things like GPT are nowhere near conscious or even sensible or even all that smart. Because not only is this haiku not that good, it is nowhere near being a haiku. [chuckle] Almost everyone knows, who's heard of the concept of a haiku, that it's a little three-line poem, five syllables, seven syllables, five syllables. So clearly what GPT-3 figured out is that it's a three-line poem. That's all it knows. It didn't get the syllable thing. Maybe it doesn't know what a syllable is. It doesn't know anything at all. All it's doing is doing correlations between different pre-existing texts. So this is not five, seven, five. So it's not a haiku, even in the dumbest sense. But what many people who don't get deeply into haiku don't realize is that there are more rules about haiku than that. Not every poem that is five, seven, five counts as a haiku by the strict original definition. It's supposed to be specifically about the seasons, or at least have seasons in the sense of winter, spring, summer, fall as a theme of the poem. And there's also like a technical requirement that I'm not gonna say exactly right.

2:14:17.2 SC: But there has to be a certain kind of punchy impact word, usually near the very end of the haiku that kind of turns, twists the meaning or what you were expecting or something like that. That's a little piece of vibrancy right at the end of the haiku that gives this short kind of poem its life. And I don't know, maybe GPT-3 thought that funny was that kind of word. But I think by most of our standards, it doesn't really qualify. If you wanna read the other poems, please check out on Patreon. And if you're not a Patreon supporter, this would be a good motivation. You wanna read these other GPT-3 haikus, let me tell you. Chris asks, "What would you say are the starting points for your world views/philosophy/science that are themselves not based on something else? And how do you justify these, as the most self-consistent set of hypotheses or something else?" So yeah, as I said before, it's more about self-consistency than about foundationalism. I talk about this in The Big Picture. There was always this idea back many years ago in philosophy that we would find unimpeachable truths. We would find foundational axioms, postulates, whatever you want to call them, on the basis of which you could then just build up a whole set of completely true, absolutely believable, rigorously demonstrated statements.

2:15:43.9 SC: And that's just not how the world works. You can do it in math. You can say, "Alright, here are some axioms. We're gonna derive the implications of these axioms." But in physics or in science, you wanna know whether the axioms are true. You wanna choose the axioms or the foundational principles that actually accurately describe the world. And that process is never 100% reliable. There's always some fuzziness there. Again, that's the theme of this AMA. So that's okay. So I don't know. I had not even tried really to axiomatise my own world view in any way. Off the top of my head, I would say the two things I kind of start with are the idea that the world is intelligible. In other words, that there are patterns out there. There are ways to make sense of what happens in the world. It is not all completely random, nonsensical, meaningless. And the other is that our sense data, our input tells us something true about the world. Now, that's a trickier one, because what it tells us is clearly not the unvarnished truth. And it is really not the whole truth. But I'm imagining that it is not completely false either, that we're not being tricked by an evil demon or anything like that.

2:17:00.9 SC: And so we try to build up systems of understanding how the world works based from these starting points and see how it goes. And it is never perfect certainty that we achieve. But it's a way of getting through the world. And so it's more of a pragmatic/practical justification. It's what works by its own lights. It's a self-consistent set of ideas that does what it aims to do. That's why I think that the basic scientific hypothetical inductive model works pretty well. Joey says, "I've been thinking that our society is really bad for people with serious mental illnesses, such as schizophrenia, as we believe in people are rational and give them a degree of self-agency... " Sorry, "As in we believe people are rational and give them a degree of self-agency. But for people who behave irrationally, many of our social safety nets break down. Other than a benevolent overlord, what do you think would make the world better?" Well, I don't... This is not one of those things where I have a clear program to advocate for or anything like that. But I agree that we have issues in society with people with serious mental illnesses. But this is kind of just, I think, part of a set of problems we have in our society about people who are not well-off, who are not empowered in various ways to make a healthy life for themselves.

2:18:26.9 SC: We do have this idea, and again, it's an important idea, even though it's a fuzzy one, that some people are responsible, that they can decide for themselves, that they can vote and they could drive cars or whatever, and they can basically form their own lives up to the constraints of the real world in which they live. And other people are not responsible, they must be taken care of, they are not to be held responsible or blameful for things that they do, because of either mental illness or they're too young or whatever. And as I said before, these categories make sense. They work. And they're important, even if we cannot define them directly. I do think about helping people who are mentally ill, that we can do a much better job. I came from Los Angeles where there's a lot of homeless people around. I believe there's also lots of homeless people here in Baltimore. Many of them are either drug addicts or mentally ill in some way. And I would like to help them in some way. I think that as a society, we're certainly wealthy and powerful enough to give everyone a home, to not let there be homeless people. And we're making a choice as a society to let this issue continue, 'cause we don't care that much. And I think we should care more.

2:19:44.8 SC: What should I actually do about it in a practical, political level? I don't know. I'm not that optimistic that way. And I really don't have anything to offer. But again, happy to listen to other people's ideas. Rob Butler says, "If microscopic objects behave classically, where does the quantum world end and the microscopic world begin? If these two descriptions, small, quantum and large classical act so differently in our reality, why do we need a unifying theory?" Well, we don't need a unifying theory theory for classical and quantum, because we have one. It's called quantum mechanics. Classical mechanics is the limit of quantum mechanics. So that's been completely unified. That's a little bit too glib. The set of situations under which objects in the world are well approximated by the classical limit is a little bit tricky. There's still a little bit of work to be done there to make that perfectly clear. That's part of what I did in my paper not too long ago with Ashmeet Singh on quantum mereology, how is it possible to divide up the quantum world into classical systems plus environments? But still there's no quest to unify classical mechanics and quantum mechanics. Classical mechanics is a proper subset of quantum mechanics.

2:21:00.2 SC: But one other thing to say about that is that people sometimes say quantum mechanics begins to be relevant when systems are small or something like that. And that's not exactly, precisely, rigorously right. The right thing to say is, quantum mechanics is always true, but there is also a sub-set of circumstances under which classical mechanics is a good approximation. And there's a subset of circumstances under which classical mechanics is not a good approximation. And you need to invoke those parts of quantum mechanics that are not present in the classical limit. That's really what we mean when we say that quantum mechanics becomes important in the small scale microscopic world. Brent Meeker says, "In the solo episode on finding gravity within quantum mechanics, Mindscape 63, you speculated that there are only finite many degrees of freedom in a given volume. Wouldn't this imply that there are also only finitely many possible states, and so there will be a smallest non-zero probability of any possible event?" So no, that's not quite right. So yes, to the first half of that sentence, but not to the second half. So finitely many possible states, that is in fact what is implied by imagining only finite number of degrees of freedom. But think about how quantum mechanics works. States are vectors.

2:22:16.9 SC: Even if you only have two states like a spin, could be spin up or spin down, you also have superpositions of spin up and spin down. And that superposition is some complex number times spin up plus some complex number times spin down. And the probability of observing spin up or spin down is the square of that complex number. Now, that complex number can be anything between zero and one. It can be arbitrarily close to one or arbitrarily close to zero, that has nothing to do with the number of states. So the probability can be as small as you want, even though there's only two states. And the same thing is true if you have many more than two, but still some finite number. Greg says, "An omniscient super being gives you one and an only one question about the true physical nature of the universe, which they will answer correctly. But it has to be a yes or no question. You know the answer will be correct and absolutely true. What do you ask?" So I'm gonna weasel out of this question a little bit. I'm not gonna answer it directly, because what I wanna say is, I kind of don't think that these questions are the best way of thinking about knowledge or the quest for knowledge, because there's always this question of what would you ask, one question would you have answered by God or whatever about physics and so forth?

2:23:31.6 SC: But the thing is, without knowing how the world works at the fundamental level, we don't even know which questions to ask. You know if Isaac Newton had been asked that question, he might have had some ideas about what he wanted to know. But he wasn't gonna discover quantum mechanics or general relativity by asking one question of God, even if God was in an answerable mood, because he doesn't know what to ask, what is the issue to be addressed. And I think that the same thing is true. If our goal is, "I wanna know the once and for all laws of physics," that is a goal that I have, or how the universe began, but if I ask the question how the universe began, presumably, that would require a long elaborate discussion of the true physical theory of the universe and what it was actually implying about the moment of the beginning of the university, if there indeed was any such moment. So it just doesn't take the form of a yes or no question. So anyway, I don't think that this oracle kind of model where you imagine asking a question and getting a once and for all reliable answer is a very good one to think about in the case of science. However, just to sort of try to do my best, it would be nice to know whether the universe lasted forever or not, whether it had a beginning or not.

2:24:48.1 SC: But again, I foresee that the answer would be, "Well, maybe time is not fundamental." So it's not even a yes or no question. That's why I think that these questions are sort of hard to imagine getting useful answers to. Paul Torek says, "And you talk on the arrow of time in causal networks. You say if you want to draw causal network arrows that only go one way in time, you only get right answers drawing them from past to future. That's a pretty big if. If there's more to say, what if someone doesn't want to do that?" And then in parentheses, "I simplified and hopefully not oversimplified the quote to save time." So maybe you anticipated this being the answer, but you did oversimplify the quote. Sorry about that. The quote in the talk was some... I forget exactly it myself. But the point of it was saying not if you want to have arrows that only go one way in time, but if you wanna have arrows that only go from one moment to the subsequent or preceding moment. In other words, if you don't want the arrows in your causal diagram of the whole history of the universe to spread out over all moments of time.

2:25:50.7 SC: So the thing about this whole causal network analysis and the project that I'm trying to do, connecting that to fundamental physics, is that there are no arrows at the level of fundamental physics. There's just a pattern that is unbreakable and perfectly reversible from past to future, according to our best current understanding. The arrows and causality only come about in the macroscopic emergent world. They're a useful approximation. So it's 100% legitimate for me to ask for features of my useful approximation that are useful to me. And it is possible, according to the rules of the game, to draw causal arrows from one moment of time to the next moment of time, but also in a network that involves arrows from the Big Bang to that moment that do not pass through all the subsequent moments. But that's kind of useless. That's not really what I'm looking for. It's like saying, "I can explain any effect at all, by saying the cause of it is the initial conditions of the universe plus the laws of physics." But it's even less useful than that, because I'm saying I can explain what happens next by what happens now, plus independently what's happening at the Big Bang. It's just not a very useful set of explanations.

2:27:05.9 SC: The nice thing is that the way the laws of physics work is that I don't have to have causal networks coming from the Big Bang or from the future or anything like that. I can invent a sensible causal network where the arrows just go from one moment of time to the next. But I can only do that going from one moment to the next future moment, not to the previous moment in the past. And that's a non-trivial statement. Patricia Paulson says, "I watched your lectures on Dark Matter, Dark Energy from the great courses. They were recorded in 2007 before the LHC was operational and the Higgs was discovered. I believe you said something like how you hoped in five years or so there would be some big discoveries regarding dark matter and dark energy. Fifteen years later, would you have lots more discoveries to expound upon in a new series of lectures, or would it be pretty much the same as 2007?" So my guess is... I don't remember exactly what I said. I'm certainly willing to believe that I said I was hopeful in five years to get big discoveries regarding dark matter, 'cause I was hopeful of that in 2007. It has not turned out to be true. My hopes were not fulfilled. I doubt that I said that there would be big discoveries regarding dark energy, 'cause I think that the discovery in 1998 that the universe is accelerating is perfectly well explained by the cosmological constant.

2:28:23.5 SC: And if that's true, there are no more empirical discoveries to be made about that. We need a better theory to explain the value of the cosmological constant. But what the cosmological constant is, is perfectly well understood. We don't need to do more. If the dark energy were something dynamical, or not just a cosmological constant, then there would be more discoveries to come. But I think that those are actually unlikely. Whereas, with dark matter, we don't know what it is. We don't have an overwhelming plausible candidate like we do for dark energy. So there, I was hopeful that we would discover something. And five years was a reasonable sort of 50-50 credence. But we turned out on the sad side of the 50-50. And it's part of a problem that is plaguing all of fundamental physics. Whether it's particle physics, cosmology or whatever, we haven't had any new truly surprising discoveries. So the only updates really for the 2007 lectures would be some details about astrophysics. We know a little... Certainly more about large-scale structure in surveys and so forth. We discovered the Higgs boson 10 years ago, 10 years ago as I'm recording this. And so that would be something to note. But it hasn't really fed in, in any direct way to knowing what the dark matter is or the dark energy.

2:29:37.1 SC: And so we have no direct evidence for what the dark matter is. But again, that's something that could come within the next five years. But I'm less likely to predict that now. So in 2007, I was happy to give a 50% chance that something good would happen in the next five years. But that doesn't mean that there's still a 50% chance that something will happen in the next five years, or the next five years, because in the space of all possible theories, there is a certain amount of phase space for, it's gonna take a really long time, maybe forever before you get direct evidence about the dark matter. And that credence has grown since 2007. I still think that it's likely that in the next, I don't know, 20 years, let's say, it becomes harder and harder to push our experimental limits. It's more technology, 'cause as the limits get further away, it's harder to get there. But I'm still hopeful that in my lifetime we will find some direct evidence for dark matter. But I could be wrong, you never know.

2:30:32.9 SC: Phil says, quantum mechanics claims that one cannot predict with certainty when a radioactive atom will decay or even given a complete understanding of the laws of physics as well as complete knowledge of the state of the universe. How can quantum mechanics make this claim, given that we don't yet have a complete understanding of the laws of physics, nor a complete knowledge of the state of the universe?" Well, it's the difference between the claim being made by quantum mechanics and the claim being made by reality. So in quantum mechanics, we know the laws of physics governing quantum mechanics. It's presumed that you're talking about quantum mechanics. Quantum mechanics is a well-defined theory. And it says here at the laws of physics. You don't have to believe them. They might not be right. But they're the laws that are the quantum laws. And so, according to the rules of quantum mechanics, you cannot predict with certainty when a radioactive atom will decay. So if we discover some way in the future that we can predict with certainty when a radioactive atom will decay, it will be because quantum mechanics is wrong, not because we didn't understand what quantum mechanics was saying.

2:31:39.5 SC: Joel Curtis says, where do you stand on the merits of the rubber sheet analogy to illustrate space-time curvature? I see a lot of people denigrate the analogy. But I think it does a fair job of striking a balance between making idea accessible and not sending enough people away with entirely the wrong idea." Or sorry, not sending people away with entirely the wrong idea. Yeah, I think it's fine, the rubber sheet analogy. I think analogies are an important part of explaining scientific concepts. Some analogies are better than others. Every analogy has the problem that parts of the analogy are supposed to tell you something about the thing being analogised, and parts that are supposed to be ignored. So when you have the rubber sheet analogy, like put a bowling ball on a rubber sheet and you say, "Look, the rubber is now being warped and curved. And that's kind of like the ball is warping space-time." The problem with that is that you're using the force of gravity to pull down the ball, and that's warping the sheet, so there's a directionality downward. None of that is true in space-time, where the earth or the sun just sit there and warp space-time around it, and there's no gravity pulling them down. It's their warping that is gravity.

2:32:46.7 SC: So there are aspects of the analogy that are not actually illuminating real features of the world. That's okay, as long as you get that, as long you explain that as part of the analogy. There are other analogies which I don't like, because it is very difficult to let go of the misleading parts of the analogy. So I'm not a big fan of the rubber balloon analogy for the expansion of the universe, which is a little bit different than the rubber sheet analogy for the curvature of space-time. And the reason why is the balloon analogy has two things. You take a... Two things wrong with it. You take a balloon, you put some dots on it, you blow it up and you see that the balloon is expanding, the dots are moving apart from each other. That's kind of like the expansion of the universe and the dots are galaxies. That's the analogy. Two very big problems with that. One is that the dots grow when the balloon grows. And galaxies do not grow with the expansion of the universe. They just get further apart. So that's false. And even worse, the balloon has an inside and an outside. The balloon is expanding into the space around it. And so no matter how hard you try, when you use that analogy, you're giving people the impression that the universe is expanding into something. And it's not.

2:34:02.9 SC: So to me, even though it's a fine analogy in the sense that it plays by the rules, some aspects are illuminating aspects of the thing being analogised and some are not. It's a bad analogy because it's really hard to undo the damage of the inapplicable aspects of the analogy. Whereas the rubber sheet analogy for space-time curvature, I think it's pretty clear to explain to people what aspects are important and which ones are not. Walter White asks a priority question, "Does the block universe imply eternal life or immortality in any sense? And if yes, does it mean our life repeats itself over and over again. Since the world lines are a finite length, I couldn't imagine any other way in which we live forever. Or am I misunderstanding the concept?" So I think I'm gonna have to apologize, Walter. I know this is a priority question, but I don't get it. So the answer is no, the block universe does not imply eternal life in any sense at all. So I don't know why it would. The block universe just means we treat every moment of time as being equally real. So an important caveat there is that because we live in time, people tend to think... People tend to attach an extra word of real at the same time or real simultaneously or something like that. That's not what the block universe is saying, different moments of time are real at different moments of time. They're just equally real.

2:35:23.7 SC: So there's no immortality. Immortality means you live through all the moments of time. And in a block universe, you don't. So there's no reason at all, I think, to connect those two ideas. Eric Davigi says, "Possibly a dumb question, but I feel this is a safe zone for that." Yes it is. I might not answer it, but you're certainly welcome to ask any dumb questions. "Why do objects bend space-time? I get that they do, but I mean, why?" [chuckle] So I understand. And there's two kind of approaches to answering a question like this. One is, just because that's what the laws of physics say. And that's a perfectly legitimate answer. It's not wrong. You can imagine alternate worlds, other possible worlds where objects did not curve space-time. So we happen to live in a world where objects curve space-time, according to the laws of physics. That's a perfectly legitimate answer. There's another level of answer which sort of makes some assumptions and then builds conclusions on top of those assumptions. So the best way to do it is something that... Is probably through like the action principle. The action principle says that in ordinary classical mechanics, we have some trajectory for a ball rolling down a hill or a planet going around the sun.

2:36:38.3 SC: It has a quantity you can define called the action, which has to do with the kinetic energy minus the potential energy over the path as a whole. And so basically you can say, "Why do planets orbit the sun in ellipses?" Well, you can say it's because of an inverse square law force. But alternatively, you can say that is the path that minimizes the action. And the action is a competition between trying to... Well, you wanna minimize the action. The action is kinetic energy minus potential energy. And so you want small kinetic energy, but large potential energy. And so you might say, "Well, small kinetic energy just don't move." But you wanna actually do a trajectory that starts somewhere and ends somewhere, so you have to move a little bit. So you need some kinetic energy. And then you want to balance that with potential energy, and the real motion around an ellipse is what does that. There's a similar line of reasoning for curved space-time that was invented by Hilbert in consultation with Einstein. There's a way of thinking about the action for space-time. And basically there's a contribution to the action from matter, from stuff, from energy and momentum. And there's another contribution from the curvature of space-time. And you wanna minimize the overall amount of action. And the way to do that is to have space-time curve a little bit in response to the energy and momentum inside.

2:38:02.4 SC: Now, everything I just said there is true, unless I made a mistake, which is always possible. But does it explain why matter curves space-time? I don't know. It depends. I mean, why is the action principle true? Why does the action take that form? Doesn't explain any of those things. We're just assuming from the start that they're true. So whether that's satisfying to you or not, I am ill equipped to judge. Gary Upshaw says, "Throughout human history, humans have accessed new forms of energy sources, animal muscle, wood, wind, coal, oil, etcetera, nuclear fission." So anyway, I'm not gonna read the whole question 'cause it's long. Remember, shorter questions are the ones that get answered. But I'm gonna just paraphrase this one. Gary is saying, "We're soon gonna invent nuclear fusion, have a huge amount of energy. New energy sources are generally associated with depleting the resources of the planet and sort of doing environmental catastrophes. So are we worried that fusion is gonna do that?" So I don't think so, for two reasons. Number one, I'm not at all convinced that nuclear fusion is imminent. I would like it to be. I certainly hope it is. But the technological barriers are severe. And we've been trying to do it for a long time and haven't yet. So some caution is in order.

2:39:13.5 SC: The other one is, I don't think you can simply say that when we get new energy phases, human population has increased and we've been accessing the earth's resources better. It's a much more complicated story than that. So I can certainly imagine that this particular kind of new energy source, nuclear fusion, would help us not deplete other resources that the planet has. We don't need to dig for fossil fuels as much if we have unlimited fusion power. So I just think that the question's a little bit more complicated. And it might. It might give us new ways to do damage to the planet. But I don't think that's a necessary or inevitable consequence. Richard Graf says, "I'm 62 and aware of the valuable perspective that comes with those in later years. However, I've come to believe that our nation would benefit from an upper limit somewhere before 60 on the age a person can be elected a US President, Senator, etcetera, to continue to serve on the Supreme Court or whatever. What are your thoughts on this idea?" So I don't love the idea, Richard. I don't think that there should be age limits, because I think that some people who are older are perfectly capable of being really good at these jobs. And some are not. I would rather judge people on their actual capacities rather than their age or something like that.

2:40:31.9 SC: I do think that term limits for Supreme Court justices make absolute 100% sense. There's zero reason why Supreme Court Justices should have lifetime terms. It really makes things worse because in the current really dopey system that we have for choosing the Supreme Court, there is this huge incentive for presidents and senators to find ideologically compatible Supreme Court Justices who are really young, because they're gonna serve for the rest of their lives. If there was just a term, eight years or 10 years or whatever, that you serve on the Supreme Court, then that would be... There'd be no incentive to find young people. You would just try to find the best people. And that just makes enormously more sense to me. There's no more political freedom that you have from having a guaranteed lifetime term than you do from having a guaranteed 10-year term or whatever. But I don't care what age you are at that. Having said all that, there is absolutely a problem in the current US system with a gerontocracy. There are too many of our leaders who are really old, especially in the Democratic Party, by the way. Republican Party has much more churn in its leadership and a lot more young people. But the people leading the Democratic Party have been around for decades now. And I don't think that's a healthy thing.

2:41:49.5 SC: Any individual, one person can be as old as you like and still be very good at their job. But there's a tendency with the way that apportionment works, etcetera, for people to be elected into office, especially in the Senate and just keep getting elected forever. And that... Why would they leave? It's a cushy job with a lot of perks. So we end up with really old people. And that's not necessarily good. And I would like to come up with ways to fix that. Mr. Foo says, "Priority question. With relation to Many-Worlds, do the other worlds have an XYZ coordinate within space, or do they exist in other locations that cannot be defined by distance from Earth? So no, they do not have an XYZ coordinate within space, nor do they have an existence in another location at all. You should not think of the worlds as existing in some other space, as having location in any space at all. They exist simultaneously. That's the thing to say. But they don't have locations in space. The idea of location in space is something that exists within each world. Space is an emergent concept inside the worlds. It's not that the worlds are embedded in some bigger space.

2:43:03.4 SC: Graham Clark says, "In his book, The Revolt of the Public, Martin Gurri talks a bit about science and using climate science as an example, portrays the institution as acting like a cartel defending its researchers and findings against any opposing views, however valid. And with a peer review process hopelessly inadequate in a time of massive data sets and esoteric research areas, does any of this ring true to you in your research areas or elsewhere?" So I have to confess, I've not read the book. I know nothing about what Gurri is trying to say. So I cannot pretend to actually be commenting on what Martin Gurri said or the book. I can respond to the paraphrase here in this question. And for the most part, no, this does not sound right. It sounds like a sort of overly alarmist hot take rather than an actual nuanced view of how science works. There are aspects of these critiques that absolutely makes sense. Of course, scientists defend their own views, like most people do. If they didn't wanna defend those views, they would probably have other views.

2:44:08.2 SC: I think that scientists are more willing than many sets of people to imagine alternative views and to think about them, especially if you say defend finding against opposing views, however valid. One of the great things about science is that even though scientists will defend their views for a long time, once an obviously more valid view comes along, they will change their minds. That's one of the great things about science. It's easiest to do when the new view is backed up by an experiment, by data. That's when people will really change their minds very rapidly. But even a really knocked down theoretical argument will convince people to change their minds. What you have very often is unconvincing theoretical arguments that do not convince people to change their minds, even if they might be right. Because you can have an argument... Let's think about the foundations of quantum mechanics. Let's think about spontaneous collapse theories. No, actually forget about those. Yeah, well, alright, let's think that spontaneous collapse theories versus Many-Worlds. Both of those have disadvantages. Many-Worlds has the disadvantage that there are Many-Worlds, and that it's a very different metaphysical view of reality than we ordinarily have.

2:45:21.6 SC: And even if that's perfectly fine, I think it's also fine to be methodologically conservative and say, "Look, this is such a huge leap from our ordinary way of thinking that we should be cautious about it." Whereas, something like spontaneous collapse theories, you're doing tremendous violence to the Schrödinger equation, which is a very elegant equation by just sticking in there some arbitrary, random collapse idea. So there's always pluses and minuses. And different people will have different feelings about which pluses and minuses matter and which ones will eventually be understood better. So that's disagreement that scientists just put up with. And it's not a matter of one camp or the other not listening to reason. It's just that they have different very reasonable ideas about what will eventually turn out to be the case. Peer review is certainly imperfect in many ways. But to call it hopelessly inadequate is hopelessly wrong. This just seems like cheap shots rather than carefully thinking about how science works, which makes me sad. Because there are legitimate critiques about how science works. And I think that it's perfectly fair to think that scientists will go too far in supporting the kinds of research programs they approve of, and not far enough in allowing other alternative research programs to flourish.

2:46:40.2 SC: That's something I've already said earlier in the AMA. I don't think that climate science is an especially good example, because it's an unrepresentative example because of all the political pressures around it. And those political pressures both convince a whole bunch of people of really dopey things, like the climate is not changing or it's not because of human beings. But they also, because scientists themselves are human, they introduce non-scientific considerations into how scientists talk about it. The same thing is true for evolutionary biology, especially back in the days when creationism was a big worry. A lot of evolutionary biologists would not want to allow certain kinds of critiques of evolutionary biology, because they opened the door to creationism. And I never went along with that. I'm like, "Let in whatever critiques you want. If they're bad critiques, then respond to them." And I think that ideally in a perfect position, the same thing would be true for climate science. And it largely is, but there is some correctness to the worry that climate scientists are...

2:47:49.4 SC: They know that the climate is getting worse, that it's because of human beings, and so they really want to stamp out criticisms of methodologies that get there, even though some of those criticisms might have a point, even if the overall conclusion is not changed, some of the details might be up for critique in some interesting way. So there are real critiques there, but anyway, within the version that is presented in his paraphrase, I think that that's just going way too far in not especially nuanced, careful direction. Late Night Linux says the scientific method was devised by white men, same with math. Some people say this makes them inherently biased, and possibly even racist. Is this something that you've thought about? I haven't thought about it a lot, but also I don't know a lot of people who say that. So I think that there might be people in the world who say things like that, but there are very few of them, and certainly not very influential ones, it is close to a critique that is made by more people and more influential ones.

2:48:50.2 SC: Namely that science and math, even if they are perfectly objective, have been used in the cause of biased racist points view or social structures or whatever, and that's just true. That's not to say that the science or the math are biased or racist, they're just science and math, but they can be put to racist or biased purposes, they can be used misused in that way. And that's something we should think about. Also, it's not even true that they were devised by white men, there were a lot of white men had very important roles to play, but a lot of people who are not white or not men also had important roles to play, so I don't wanna silence them either. Joe Moskowitz says, I was watching you something deeply hidden talk at Google, and something too go lose in my brain and Many-World suddenly seem more reasonable. The slide with the expression for the dead and alive Schrödinger's cat multiplied by the environment and the observer because of the universal wave function seems to have been the exact tipping point, even though I've seen that before.

2:49:52.9 SC: I'm highly suspicious. So 50 years in my opinion shifting in a few seconds. Any advice? So I like this a question because it's actually not a question about quantum mechanics, you thought it was gonna be a question about quantum mechanics, didn't you? But it's a question about changing your mind and... Yeah, I don't think you should be suspicious, I get why you're suspicious, that you had a point of view for a very long time and it all seemed perfectly sensible and you were gonna stick to it and sudden you switched, but what I really think is that it's less sudden than you imagine, because I think that when we have beliefs, when we have points of view or credences or whatever, we have sort of our conscious level beliefs, but there's a lot going on beneath the surface, and it's an example, Jennifer Ouellette, my wife, talks about this in a talk that she gave, in an article, she's written about a plateau kind of effect when it comes to people's changing beliefs, and it comes from an example of when you cook... When you slow, slow cook, pulled pork on... It's not pulled when you're cooking it, you're cooking a pork shoulder or, whatever.

2:50:58.5 SC: Low and slow over barbecue. And if you do that and you put a thermometer in there and you plot temperature versus time, what happens is as you begin to cook over a very low heat, the pork shoulder, the temperature goes up almost linearly and then it plateaus, remains more or less constant, even though you're still heating it up, and then after a while, the plateau stops and it starts heating again, and pit masters, barbecue masters know about this, and they know that what's actually going on is when that plateau hits, there's been a transition where it is easier for the pork shoulder to just expel vapor, expel water vapor and lose heat that way, or I suppose gain heat that way. I'm not sure what it's doing. It's not gaining temperature, but it's... Rather than gaining temperature in the pork, it is instead expelling hot water vapor into the barbecue pit, and that's bad because you want your pork shoulder to be moist, so when the plateau first hits, you open up the grill, wrap it in aluminum foil or whatever to keep the moisture in and then put it back.

2:52:08.6 SC: Why am I saying all this? Because if the only thing you knew was the temperature, that that's the only variable that you were keeping track of, you would say, Well, the temperature was going up, and then it was constant, and then it started going up again, but beneath the surface, something is changing while the temperature is constant, there's another variable, you are not keeping track of, the moisture, the level of water content in the pork shoulder. So I think the same thing happens all the time with our beliefs, which is that we think our beliefs are constant, but there's a set of sub-beliefs, is what I'm trying to say, lying beneath the surface that change gradually. You get more information, you put pieces of information together, slowly, gradually it builds up, and then what it appears to you like is a sudden phase transition. You're like, Oh, yesterday, I believed this. Today, I believe this. Why do I have this sudden transition?

2:53:08.2 SC: I think that usually the answer is because your mind has been changing, but in invisible ways, in ways that do not show up on the surface, which by the way, is why I think that there's a lot of issues involved with studies of how to change people's minds, because they work on short time scales, they will present something to you and ask, have you changed your mind right now, and maybe the answer is no, but maybe that thing that's been presented to you will linger on for days or months or years and have an effect on you changing your mind years later, which the psychology experiment will never capture, so something to keep in mind when we think about why people believe things and how those beliefs change.

2:53:46.8 SC: Simon Reynolds says, have you ever tried to enter a friendly wager with another scientist, and could you consider that as a good way to help people consider their decisions more carefully? Yeah, I absolutely have, and I almost always win because I don't enter into bets that I think that I have a decent possibility of losing. You never know for sure, of course, everyone thinks that, but I don't enter into many 50-50 bets.

2:54:11.5 SC: I won a bet that some people know about with Brian Schmidt, who was my office mate in grad school and later won the Nobel Prize for helping lead the team that discovered the accelerating universe. Back when we were both in grad school, he was the cynical observer and I was the starry-eyed theorist, and we bet about whether or not, astronomers in the next 20 years would be able to measure the density parameter of the universe, 'cause astronomers have been trying to measure the amount of matter in the universe, and it was hard and they didn't get the answers that the theories predicted, etcetera. Now, when Brian and his friends discovered the acceleration of the universe and therefore the dark energy that fixed the problem, then we knew what the extra energy density of the universe was. So on the one hand, I won the bet 'cause I said, we would figure it out, and he said we never would, but he won the Nobel Prize, so it's a win-win bet really, from all points of view. P. Walter says in your 2020 Royal Institution on the multiverse, you explain to how linear tracks that appear in cloud chambers only seem to infer a particle-like cause rather than a wave-like cause.

2:55:17.7 SC: The talk was excellent, and helped clarify what Ken and Kempi known about the multi-verse, but I'm still confused about why there are linear cloud chamber tracks, could you have another go at explaining this phenomenon? Sure. Actually, I think that this is something where as far as I recall, I did not go into detail about this in something deeply hidden in my book, but it's worth going into detail... Maybe I should have, and I regret not doing that. This question of why, if you just saw the Schrödinger equation, you have a nucleus that decays, you can solve it for what the radiating particle will look like, what the wave function of the radiating particle will look like, and roughly speaking, it's a spherical wave. The particle that is emitted is emitted equally, it's wave function looks more or less the same in every direction. That's not gonna be exactly true, but that's a pretty good approximation, but it's not what you see when you measure it, so you do that inside a cloud chamber.

2:55:42.9 SC: What do you see? A line? Not a big spherical cloud. Why do you see that? Well, you're gonna guess correctly the answer, right? Is Because of decoherence and things like that. So what happens is there is a spherical wave function that is produced, it moves out in every direction, and if you subscribe to the Many-World's way of talking about this stuff, it interacts and becomes entangled with the particles around it in the cloud chamber, but the point is that as soon as it interacts with one particle in the cloud chamber, then the universe splits into the part of the universe where it interacted with that one particle and the part where it didn't.

2:56:48.2 SC: And in the branch of the the universe where it interacted with that one particle, the rest of the wave function, the rest of the spherical sphere, the cloud of wave function that was emitted from the nucleus, the rest of it is no longer there in that universe.

2:57:05.2 SC: It's in the other universes. There's one universe where the particle was not interacting with that particular part of the cloud chamber, and then the rest of the wave function says it did. Okay, so in the branch where it interacted with that particle and became entangled, the wave function of the radiated particle is no longer spherical. Now it is localized near the particle that it interacted with, okay, because that's where it hit it, and so that little wave packet now of the radiated particle that interacted with the cloud chamber keeps going in a straight line and is no longer a sphere, it's just a little localized wave packet, and then it just keeps interacting with other particles after the fact, and it forms a line.

2:57:46.7 SC: The rest of the wave function, which is now like a spear minus a little hole, that keeps interacting, and if it interacts with one particle in some other location, then it will again branch the wave function into a little wave packet moving in that direction. Plus the rest of the universe and so on and so on. So you get many, many universes, each one of which has each particle being radiated, describing a straight line or a curve line, if you're in a magnetic field inside the cloud chamber, so that's why waves begin to look like particles in quantum mechanics once you start observing them. Daniel Fox says, I recently had a minor stroke while running, no apparent lasting damage, however, no one can say exactly what caused the occlusion, it just seems like a random thing without any of the usual health or behavioral indicators. I've been a big ultra distance runner for years, and it's been a big part of my identity.

2:58:47.8 SC: The question is, how would you approach the decision of weather and to what extent you would continue to run. So preface this by saying I'm not a doctor. I have a PhD, but I am not a medical doctor. I am not the one you should ask this question to, so there's a methodological question here that I can talk about, but I'm not gonna give advice about how to deal with your health, the methodological question is, it sounds like the issue is that something bad has happened, you had a minor stroke while running, and you don't know whether it is associated with the running or not. So if you did know, right? Then there's still a question, because you say there's a probability, a risk of having a little stroke while I'm running, but I like running and running has other good benefits and how do I weigh that risk? And that's a perfectly legitimate set of questions to ask, and I don't exactly know what the right answer is. Different people's judgments will be different. That's completely fine. The difficulty, I take it in this question is that you don't even know if the stroke had anything to do with the running or if it was just coincidental.

2:59:47.7 SC: If you thought that there's a serious possibility of having a stroke, then stop running. There are other things you can do, but you're in this epistemological, tricky position, 'cause you don't know. And that's really, really hard, honestly. That's all I have to say. I do think that it makes sense to be cautious to really do research about what are the reasons why people have strokes and do any of those reasons have to do with running? And the best you can do in situations of uncertainty like this is minimize your uncertainty. There will always be some uncertainty, but you can try to minimize it, so you can say, Look at all the data or hopefully look at the data that has already been analyzed by somebody else, but do runners have more strokes? Do people who run tend to have their strokes while running? Even if you can't pinpoint a relationship causally in yourself and your body, you can at least statistically look at the relationship in other people's experiences, and if you find that, yeah, people who are distance runners tend to have more strokes or people who are prone to strokes and are distance runners are more in greater jeopardy because of that, then I think it's reasonable to conclude that you might be in more risk than you otherwise would be, and to take that seriously.

3:01:04.2 SC: So I guess the only thing I'm saying is, even in the absence of a direct mechanism to explain where the danger is coming from, you can still use indirect methods to see that there could be a danger there and take it into account. Kathy Seager says, a question about consciousness, referring to the Mind Chat discussion you participated in in May. So by the way, Mind Chat, not Mindscape. Mind Chat is a podcast video podcast from Philip Goff, previous Mindscape guest, and Keith Frankish. And they're both philosophers who care about consciousness and Philip, of course, is a panpsychist, and Keith is a physicalist/illusionist. The way that he likes to talk about consciousness is that it's just an illusion. That's not my preferred way of talking about it, but in fact, push come to shove, we're pretty close on our points of view. So the two of them disagree with each other, but they do in a friendly way, and it's worth listening to.

3:01:58.0 SC: So I did a couple of episodes with them, one just by myself, one with Barry Loewer, who's a philosopher at Rutgers. Anyway, Kathy says, I'm not going along with Philip Goff approach of panpsychism and stating the biologics and principal physics therefore let's skip biology and mess around with the core theory. As soon as particles/fields make up neurons, neurotransmitters, etcetera, from there on it may be better to talk in biological terms instead of skipping biology all together in the discussion. So I think that... I'm almost on Philip's side here for this one, so let me explain what I mean by that.

3:02:38.7 SC: Certainly, if the question was, how do we explain consciousness, then talking about biology and neuroscience and neurotransmitters is absolutely necessary. Yes, you should try to do that. You cannot ignore that. But I'm not here to try to explain consciousness, I'm here to understand the fundamental laws of physics and the nature of reality. So my only role in this discussion was to talk about the plausibility of the perspective that says, in order to explain consciousness, we must change the laws of physics, the currently known laws of physics are not up to the task of completely accounting for conscious human experiences. I think that that's wrong, and I think it's wrong in two ways, Number one, I think that the laws of physics are perfectly up to that task, but number two, people who don't think that they are vastly underestimate the difficulty of changing the laws of physics, and so I think that that is a very tiny subset of the set of interesting questions surrounding consciousness, but it is a valid subset that can skip the step of biology, so that discussion of whether or not we need to change the laws of physics and the credence we should put on unspecified changes in laws of physics becoming real because of consciousness issues is a perfectly legitimate set of things to talk about, even though it's not anywhere close to, let's explain consciousness by itself.

3:04:08.1 SC: Simon Carter says, do listener numbers play a part in what topics you pick? You've mentioned before that the wine episode was low on numbers, so has that stopped you from doing more episodes around this? No, not really. I don't know exactly what my words were, but in fact, for the audio podcast listening audience, which is the vast majority, there's a tiny number of people who listen on YouTube, even though there's no video, so there you go, but the actual audio podcast numbers are actually remarkably stable from episode to episode. There's a clearly a core audience that listens to most episodes, I think that's just how it works. There will be some episodes, if I do cosmology or if I have someone like Dan Dennett on, who's very well known, then they will get a little bit of extra listeners.

3:04:50.5 SC: Roger Penrose will get you extra listeners, and the sort of slightly more out there topics will get a little bit fewer, but not enough to make a dramatic difference. We're talking about factors of the widest... So it's more of an effect in the positive direction than the negative direction, it's more that a really, really famous person will get you more listeners than a weird topic will get you fewer, so I'm not really worried about having fewer listeners with a weird topic. And aside from the few very famous people that I have, and of course, it's very famous by Mindscape listeners standards, so I remember when I first joined Wondery the podcast network, and we were choosing what would be the first episode to have for the first Wondery episode, and I said, Well, I do have Seth MacFarlane coming on, he's a fairly well-known celebrity, and they said, Yes, good, let's do that. And we did that.

3:05:52.3 SC: But as I tried to explain to them, Set MacFarlane is far more well-known in the general public than let's say Roger Penrose or Dan Dennett but he's not gonna get as good numbers in the Mindscape listenership as Roger Penrose or Dan Dennett will and he didn't. He did well, but it's not like getting Carlo Rovelli on, where you get a lot of extra people listening. So anyway, within that list of people, I think that having a very famous person will get you maybe twice the average number. Whereas, the decrement that you get from the average for having a less well-known topic is maybe 10% below average or 20% at most. It's not worth changing the topic distribution for that.

3:06:37.4 SC: But also I don't just think of individual topics, I try to make the blend of topics fun and interesting and keep people on their toes a little bit. I know that lots of listeners have their favorites, and that's cool, you can have your favorites, and basically that no one is forcing you to listen to every episode. You can listen to the ones you like, and that's great, but I'd like to slip in some episodes on topics or with people who you would not have anticipated. And hopefully, people will listen to them and be happily surprised.

3:07:06.3 SC: Or, they don't like it, they try it and don't like it, that's a feature of trying new things, that's also perfectly okay. But part of my self-appointed task is to bring in a bunch of things, maybe not the ones that you would always expect, so I continue to want to do that. Sandra Stocky says, in your solo episode on the multiverse, you mentioned that De Sitter Space is a maximum entropy state. That seems counter-intuitive. How could a highly symmetric vacuum state be high entropy, how could there be any micro-states if everything is empty?

3:07:36.1 SC: So there's two things going on here, two levels of answer to this. One is, think about it, high entropy states are always seemingly smooth, right? If I have a box of gas, if all the gas is squeezed into a corner, that's low entropy. The high entropy configuration is the equilibria where the gas is smoothly distributed throughout the box.

3:08:00.0 SC: What you're thinking of is, of course, the microstates corresponding to that macrostate are... Maybe what you're thinking of is that the microstate's individual atoms and molecules are not perfectly smoothly spread, but that's partly a classical kind of intuition that you're referring to. The De Sitter state that we're assigning a high entropy to is a quantum mechanical state. And the nature of a thermal equilibrium quantum state is a little bit different, the nature of a high entropy quantum state, it's a little bit different than a classical one because there are wave functions, not individual positions and velocities of particles. The other thing going on at a slightly more subtle level is it's quantum gravity, and we don't know the full theory of quantum gravity, so we don't know what the microstates are. So when we say the De Sitter space or a black hole or whatever, has a certain entropy, that's following Stephen Hawking's ability to associate entropy with the size of am event horizon, and it's a very robust calculation.

3:09:00.9 SC: It works at the thermodynamic level. Let's go back to the mid-19th century. We defined entropy... Clausius has defined the word long before Boltzmann came up with the definition of it in terms of micro-states. It was a phenomenal optical thermodynamic quantity that involved heat engines and things like that. Our current understanding of the entropy of a thing like De Sitter space or a black hole is similar to that phenomenological entropy. We know the macroscopic rules and we can assign an entropy to it, but we don't know the complete microscopic theory 'cause we haven't fully quantized gravity yet. So we have very strong confidence that the entropy that we associate is the correct one, but I can't tell you what all the micro-states are that go into making up that entropy. If I did, I would know a lot more about quantum gravity than I do. So in other words, the fact that De Sitter space has such a large entropy implies a certain number of micro-states, but we don't know physically how to really talk about what those micro-states are.

3:10:04.0 SC: Okay, Steven says, I have a newb question about quantum mechanics, all the strange effects like entanglement or tunnelling, how do they come about in nature, where in nature, in our universe or in our bodies, are there ever particles isolated enough, so these effects don't go away by the particles immediately interacting with someone, with something? Actually, yeah, it happens all the time, literally all the time.

3:10:26.5 SC: So consider a helium atom that has two electrons, two protons, two neutrons, and also two electrons in the ground state of the helium atom, those two electrons are very entangled with each other. Their spins are entangled with each other. They're are in what's called a singlet state, which is a superposition of spin up, spin down plus spin down, spin up, highly entangled. If you were to observe the spin of one of those particles, then you would break the entanglement, and you would excite the atom so it'd no longer be in its ground state. So in that kind of slightly trivial sense entanglement happens all the time, everywhere. It's just that once you separate the particles from each other and measure them or do something with them, you break that entanglement. So you don't notice it all the time.

3:11:15.0 SC: Tyler Ogarek, says what are your thoughts on Pascal's wager? So Pascal's wager, for those of you who don't know, to the extent that I remember it correctly, and probably not doing complete justice to Pascal, but the basic idea is, Look, maybe God exists, maybe God doesn't exist, but he lived... Pascale lived in an environment, which was completely Christian dominated, so he said, if God exists, I can believe in God and go to church and be religious, and maybe I'll go to heaven. And if God exists and I don't believe in God, maybe I will go to hell. Whereas, if God doesn't exist, it doesn't matter whether I go to church or whether I don't. Therefore, all else being equal, I should go to church and believe in God because I might get a reward from it. That's the wager, okay?

3:12:00.7 SC: This is complete crazy talk nonsense, this is a very, very bad reasoning, sorry, because it would make sense if you thought that the only two choices were atheism or the traditional Christian God, but of course there are way more choices than that. Maybe there's a God who respects your decision, if you are intellectually honest and decide to be an atheist, even though you don't believe in God, but this particular God gets really annoyed at hypocrisy and will send you to hell for believing into him for bad reasons, okay, who knows? Maybe there's another God, maybe it's a pantheon of Gods, maybe it's something completely animistic, who has nothing to do with heaven or hell. Who knows? I don't know, and I think that Pascal's wager is a very cheap way of ignoring a whole bunch of possibilities. Unless you have some probability distribution on these possibilities, you cannot accurately make that wager, as I'm sure Dr. Pascal would have been able to tell you if you've been thinking carefully about probabilities rather than about arguments for the existence of God. Amit Francis says, I'm currently finishing up grad school and in the process of interviewing for postdoc positions in the area of computational cell biophysics.

3:13:14.3 SC: You've given some of your thoughts on the plight of postdocs in past AMAs, but I'm wondering if you have any advice for me to make the most of my experience as a postdoc. I only have very vague general purpose advice. I don't know anything about cell biophysics. The vague general purpose advice is to keep in mind that just like generals are always planning for the last war, students and young scientists are always being trained for their previous position. So when you're an undergraduate, you're trained to solve problem sets, do homework, but no one cares about that when you get to grad school, when you get to grad school, the thing is... And then you're chosen for grad school on the basis of how well you did on the homework, and then who cares about the homework once you're there? Now you have to do research, so now your advisor gives you research problems and your skill set is following those research problems and solving the puzzles that your advisor sets to you. Great.

3:14:08.9 SC: And then you're hired for a postdoc and no one cares about that skill set anymore, because now you have to come up with your own research problems, and of course, that depends, if you're gonna lab, maybe you're still, as a postdoc following the agenda set by the PI, but in theoretical physics, you get to choose your own research problem, so that's a whole new skill set that you might not have had a chance to develop while being an undergraduate or even a graduate student. And then on the basis of your success as a postdoc, you're hired as a faculty member, and of course, inventing your research problems is still very important, but there's a whole new set of skills that become important, mentoring and teaching and juggling administrative duties and applying for grants, none of which you've been trained for as a postdoc. So while you're a postdoc, keep in mind that your goal, presumably is to get hired as a faculty member at some point, or as a scientist in some laboratory or something like that, and don't fall into the trap of just doing what you successfully did as a grad student, do the things that will make you employable and successful as a faculty member, and hopefully use those to have a long and fruitful career. So this involves taking initiative, this involves not waiting around. As good as many senior scientists are at being mentors, many are not.

3:15:27.7 SC: They're not trained to do it, and many kinda don't care about it, they wanna do their own research agenda, and you have to sort of go along with whatever lab you're in, but also look out for yourself, do work that you think is interesting and the rest of the world thinks is interesting. That work might not be what you were trained to do as a grad student. Think about what is going to be the most fruitful direction going forward, not what was the most fruitful direction 10 years ago when you got into this game, okay? These are hard questions to answer, but this is what you gotta do as a postdoc if you wanna keep going forward.

3:16:02.2 SC: Okay. We're getting near the end here. Don't panic, I have a few more questions. Keith says in your 2019 FQXi talk on causality in the arrow of time, you briefly contrasted the nuanced causality that relies on the arrow of time with the specific notion of causality and relativity, where it essentially just means everything moves slower than the speed of light. Do you mind elaborating a tiny bit here, is a good way of putting it that the arrow time is something that technically gets imported into relativity?

3:16:31.0 SC: No, the arrow of time does not get imported to relativity, but physicists implicitly know that there is an arrow of time when they ask questions. So when the physicists doing... A relativistic, let's say, talks about causality and you don't need... This is not even general relativity. Let's just do special relativity, so let's do Maxwell's equations, let's do electromagnetism in flat Minkowski space-time. What a physicists has in mind when they talk about causality is imagine that you have the electromagnetic field and you poke it, okay, so you by hand put in some influence that changes the electromagnetic field at a point in space-time, how do the consequences of that poking ripple out into the universe? And the answer is, they ripple out into the future light cone of that point in space-time. Well, where did the word future come from in that discussion? The answer is, you put it in.

3:17:27.2 SC: You assumed that whatever happens when you actually poke it, like putting your finger in a pond, in the water, the ripples actually do ripple out because of the thermodynamic arrow of time. So there is an equally valid solution to Maxwell's equation, where the only ripples focus in from the past on that point in space-time where you poked it, but no one cares about those solutions 'cause they're not physically relevant in the macroscopic world. So the relativistic notion of causality is ingrained in relativity. There are light cones, but the way that it is used is very, very much colored by the fact that physicists know there's an arrow of time and implicitly assume it in whatever thought experiments they are carrying out.

3:18:13.7 SC: David Wych, W-Y-C-H, says, can deep learning methods like deep neural networks and transformers really be considered tools for scientific progress when their underlying models of data generating process are not currently interpretable or meaningful? Often they don't even claim to be models or representatives of the data generating process. Well, I do think they can be considered tools for scientific progress. My analogy that I like to use, which is only... Is incomplete, it's an inadequate analogy as all analogies are, but I like to think of deep learning as sort of Tycho Brahe, the famous astronomer.

3:18:50.6 SC: Tycho, was one of the first to use telescopes to make systematic astronomical observations or... Or no, wait, actually, sorry, he didn't use telescopes, he used surveying instruments, but he did very precise, I guess, astrometric observations of the sky, telescopes were not widespread at the time, but he had an observatory, and what he would do is just look at the sky and survey it. So where are the planets as a function of time, how do they move through the sky and things like that, finding comets, what have you. Big data by the standards of the time.

3:19:24.7 SC: He collected a lot of data, he had wealthy patrons who helped him support his habit of collecting a lot of data, and what did he do with the data? Nothing very inspiring. He had a model of the solar system that wasn't very accurate or anything like that, but he didn't really contribute on the theoretical level to a vast improvement in understanding. His student, Kepler, did. Kepler was the one who realized that from Tycho's data, you could infer that the easiest explanation of what the planets are doing is that they're moving in ellipses, not in circles with epicycles, like Tycho's model did as well as Ptolemy and Copernicus and so forth. And so Kepler actually had laws, Keplers laws, and that was a true scientific understanding. So true scientific understanding is not just a correlation between bits of data that you can discover by doing a deep learning algorithm, it's a principle, it's an equation, it's a relationship that you understand why it is like that and the implications of it. So.

3:20:28.4 SC: If you had a deep learning algorithm in Tycho's time, maybe you could have plotted out the motions of the planets, but if you'd asked, okay, now, I have a hypothetical new planet, how will it move? It would have no idea, 'cause all it has done is look at the other plants. Whereas, with Kepler's laws, you can have a prediction for how other planets are gonna behave, and then Newton's laws were even better because they could talk about the perturbations on those orbits from other planets.

3:20:54.6 SC: So the true scientific understanding is not just fitting curves with data, but getting the data and finding what the curves are is a crucial part of it, so I have no trouble believing that deep learning will play a huge role in the advancement of scientific progress, but it's not the entire story. Ube Kune says, is it possible that there is another co-existing world made up of dark matter within our universe, dark matter planets, dark matter life forms, dark matter spacecraft made by those life forms, etcetera? So the short answer is probably not. It's very, very, very unlikely. And the reason why is because dark matter is dark, and that doesn't just mean that it doesn't interact with light, it doesn't interact with anything like light, or at least not very strongly.

3:21:45.6 SC: One of my favorite personal papers that I've ever written was on the possibility of dark electro-magnetism. We proposed that dark matter particles could be charged under a kind of electromagnetism, but that only talk to dark matter, not to ordinary matter. And you can do that if the force is a little bit weaker than ordinary electromagnetism and the dark matter particles are sufficiently heavy. If they're light-like electrons, then you would have noticed this a long time ago, but if they're heavy enough, then they're dilute enough in the universe that the existence of this extra dark electromagnetism isn't that noticeable. But in order to build stars and planets and spacecraft, you need the dark matter particles to be able to interact with each other and dissipate fairly effectively.

3:22:32.3 SC: And dark matter doesn't do that. We can map out where the dark matter is, and it's not in planets and stars, it's more or less diffusely spread out through the universe. So the simplest idea is that dark matter is just no interactions at all, and there are pretty good limits on how strong any possible interactions can be, that makes it very, very difficult to imagine dark atoms and so forth. But it's not impossible. So in fact, at the end of our paper on dark electromagnetism, we said, you could imagine dark atoms, and then what happens in that case is as long as the purported dark atom is made of two heavy particles, like real atoms are made of heavy particles like protons and neutrons, also light particles like electrons, but if the dark matter, both positively charged and negatively-charged particles are both heavy, then you can get away with it, then they form a dark atom that is basically like a neutral particle.

3:23:26.7 SC: Even though there is an electromagnetic force, it's more or less all invisible to you, 'cause everything is neutral, and so my new colleague here, David Kaplan at Johns Hopkins, he and his collaborators wrote some papers on dark atoms and things like that. It's very interesting stuff, and I still think that the answer is you can't go so far as to get dark planets and spacecraft, but it is an interesting intellectual exercise to imagine how far you can go. There's almost no reason to expect that that is what dark matter is like, but you can at least ask the question. Okay, the last question is from Shelton Sillyman, that might not be this person's real name, but the question is I'm curious about the makeup of your Mindscape podcast audience, what percentage are students, science, hobbyists, other scientists, etcetera? I am hopeful that a good portion of your audience is young and excited for the potential discoveries we all can make.

3:24:20.9 SC: Yeah, I don't know. It's a very good question. I left it as the last question here, because I like to think also that the audience is diverse in many ways, age, gender, distribution geographically, but realistically, I expect that the United States and other English-speaking countries are over-represented, that science-curious people are over-represented, etcetera. But otherwise, I don't know, I don't know what the age distribution is at all, I'd like to think there's lots of young people listening to the podcast, but I really don't know, maybe like there's an age sweet spot for podcast listeners that I don't know about, and maybe people who are older than that and younger than that don't listen. Not really sure about that. And Maybe podcasts are already old fashioned for teenagers, and they're on to something else, so I don't know, but I do know that... So even though I don't have an accurate representative sample, I do know that I have interacted either through Patreon or through Twitter or even in person with a bunch of people from all sorts of different backgrounds, ages, interests, things like that, so I'm very happy of the diversity that there is there in the Mindscape audience. And once again, I'm happy to all the Patreon supporters for keeping the podcast going, keeping these AMAs going. You ask great questions during the AMAs, as I always say, and I'm gonna keep saying it, so get used to it.

3:25:47.5 SC: I'm sad I can't answer all the questions. They're all worth answering. I had to pick some that I thought were the most... I could say something about or just wanted to talk about, but they're all good. Keep asking them, thank you for the support of Mindscape. Here's to another several wonderful years of more podcast episodes in the future.

4 thoughts on “AMA | July 2022”

  1. Hi Sean, The scholarship is a great idea! I am so very happy to be a part of that…and I hope the tax donation works in Canada! – Stew

  2. sean carroll hello is ther a god?is the universe cyclical infinite?is the universe always existed?

  3. Hi Prof Carroll, do u think that the conceptual, descriptive analogies and explanatory paradigms of quantum theory (eg. observer effects & measurement problems, wavefunction collapse, entanglement, superposition, uncertainty principle etc.) which are already being adapted to the decisional, cognitive and social sciences, can be applied to the field of risk studies especially in areas such as risk perception, management & governance & the correlation of risk-related (a)causalities with risk-linked consequences in complex & nonlinear systems that involve elaborate Bayesian probabilistic calculations & complicated statistical assessments?

    These pertain to quantum theory’s possible relevance to those risk research related fields which might have an increasingly pertinent bearing in shedding groundbreaking revelatory insights into the synergistic dynamics of human agency driven and machine assisted/enhanced risk perception, cognition, communication processes in tandem with the upgrading of risk mitigation methodologies and risk governance protocols with the rapid onset & ascendance of the Fourth Industrial Revolution.

    This is with specific reference to the growing assemblage of complex ensembles consisting of human cybernetic and neuromorphic interface devices that increasingly utilise a host of multiply interconnected instrumental arrays of digitally upgraded metaversal-type platforms operating under significantly sped up electronic quantum states which are now in the midst of transiting to quantum computational AI algorithmic systems in identifying, detecting, evaluating, containing and managing risks. This in turn will enable regulatory agencies to exploit the rapid paced correlational capacities and vastly expanded bandwidth capabilities of these novel innovative technologies in speedily cross-referencing and retrieving a multitude of voluminously amassed info from infotech, fintech platforms, panoptic surveillance technologies, aerospace and biomedical metadata repositories. This is just to cite a few areas of these potentially ground breaking areas of research that not only integrates but also significantly expands the scope of drawing up plausible drawbridge linkages that synthesizes the quantum sciences with more traditionally oriented risk disciplines.

  4. I get Sean’s reluctance to use “qualia”, but his discussion of colour perception was very naive. I’m sure this isn’t news to him, but there is no single wavelength of light that corresponds to perception of red; a vast range of wavelengths can be perceived as red in the right circumstances (cf that dress). Therefore perceived red floats completely free of the physical origin of the redness (redness is not a property of the world but only our perception of the world). Therefore it does seem possible that any two people might have radically different internal experiences of red. However I don’t think we could ever detect such a difference

Comments are closed.

Scroll to Top